Estate Planning: Practice Exam

¡Supera tus tareas y exámenes ahora con Quizwiz!

Which of the following are interested parties to a power of attorney? A) The principal's spouse B) The guardian ad litem C) The principal, or person granting the power D) A trustee

C) The principal, or person granting the power The parties to a power of attorney are the principal and the agent.

Jenson died this year. His will specifically bequeaths $1 million to his son, Ned, and bequeaths the residual of his estate to his wife, Nancy. At the time Jenson wrote his will, his net worth was in excess of $4 million, but at his death, his net worth had plummeted to $1,050,000. Because Nancy would only receive $50,000 (or $1,050,000 − $1,000,000) of Jenson's assets, Ned fully disclaimed his bequest 3 months after his father's death so that his mother would receive more assets. How much will Ned have to report as a taxable gift because of this disclaimer? A) $0 B) $38,000 C) $50,000 D) $1 million

A) $0 A qualified disclaimer—a disclaimer that is made in writing and filed within 9 months of the decedent's date of death—does not allow the disclaiming party to specify to whom the property will pass and does not allow the disclaiming party to benefit from the property before disclaiming his interest. If a disclaimer is qualified, the property will pass to the residual heirs of the estate, or as directed by a disclaimer clause, with no effect to the disclaiming party. In this case, Ned has no taxable gift related to this disclaimer.

Carl is the sole grantor of an irrevocable trust and did not retain any right to the trust's assets. As a lifelong bachelor, the income beneficiary of the irrevocable trust is Carl's sister, Nancy, and the remainder beneficiary of the irrevocable trust is Carl's niece, Maya. At the time of the transfer, Carl paid a gift tax of $35,000. Carl died four years later, when the value of the irrevocable trust was $1,000,000. With regard to the irrevocable trust, how much is included in Carl's gross estate? A) $0 B) $35,000 C) $345,800 D) $1,000,000

A) $0 Nothing is included in Carl's gross estate. The full fair market value of the trust is excluded from Carl's gross estate because the transfer to the trust was irrevocable, and Carl did not retain any right to the trust's assets. Furthermore, because more than three years have passed since the transfer, the gift tax paid will not be included in his gross estate.

Gene contributed $500,000 to an irrevocable trust and did not retain any right to the trust's assets. The income beneficiary of the irrevocable trust was Gene's sister, and the remainder beneficiary of the irrevocable trust was Gene's niece. At the time of the transfer, Gene paid gift tax of $35,000. Gene died four years later, when the value of the irrevocable trust was $1,200,000. With regard to the irrevocable trust, how much is included in Gene's gross estate? A) $0 B) $35,000 C) $500,000 D) $1,200,000

A) $0 Nothing is included in Gene's gross estate. The full fair market value of the trust is excluded from Gene's gross estate because the transfer to the trust was irrevocable, and Gene did not retain any right to the trust's assets. Furthermore, because more than three years have passed since the transfer, the gift tax paid will not be included in his gross estate.

Ezekiel and Cassandra make a cash gift of $120,000 to an irrevocable life insurance trust (ILIT) with Crummey provisions. The ILIT owns a whole life policy on Ezekiel's life. Their three children are beneficiaries of the trust. A few weeks later, Ezekiel dies in an auto accident. How much of the cash gift will be included in Ezekiel's gross estate? A) $0 B) $45,000 C) $15,000 D) $120,000

A) $0 This was a cash gift, not a gift of life insurance. Therefore, none of the gift will be included in Ezekiel's gross estate as the trust is irrevocable.

Bernard made a gift of $500,000 to his brother in 1997. At the time of the gift, the applicable gift tax credit was $192,800, but due to Bernard's prior taxable gifts he paid $200,000 of gift tax. When Bernard died in 2021, the applicable gift tax credit had increased to $4,625,800. At Bernard's death, what amount related to the $500,000 gift to his brother is included in his gross estate? A) $0. B) $153,000. C) $200,000. D) $500,000.

A) $0. Gift tax paid on gifts made within three years of a decedent's date of death is included in the decedent's gross estate. In this case, Bernard made the gift more than three years before his death, so $0 is included in his gross estate related to this gift. The gift will be included in Bernard's adjusted taxable gifts amount, and Bernard will get credit for the gift tax paid of $200,000.

Sharon wants to make sure that she makes full use of the applicable estate tax credit upon her death, but also wants to make sure that her husband, Oswald, has access to her property. Which of the following would you recommend? A) A Bypass Trust with Oswald as the beneficiary B) An Irrevocable Life Insurance Trust with Oswald as the beneficiary C) A Revocable Living Trust that authorizes the trustee to distribute to Oswald only under specific circumstances D) A Section 2503(b) Trust with Oswald as the beneficiary

A) A Bypass Trust with Oswald as the beneficiary A Bypass or Credit Shelter Trust would be the best option to accomplish Sharon's goals. Statement (B) is incorrect because assets in an ILIT pass outside the estate, therefore the credit would not be utilized. Statement (C) is incorrect because Oswald would not have free access to the property under those trust terms. Statement (D) is incorrect because 2503(b) trusts are only for minors.

Which one of the following transfers made this year by 85-year-old Jennifer will not be subject to the Generation Skipping Transfer Tax? A) A gift of a remainder interest in a trust just established that is paying an income interest to Jennifer. The remainderman is a grandson whose parents died in an auto accident earlier this year and before the inception of the trust. B) A transfer by Jennifer of $20,000 to a UTMA account established by her son for Jennifer's granddaughter. C) An irrevocable trust that pays income to Jennifer for 10 years and then pays the remainder to her grandniece who is 65 years old. D) An irrevocable trust that pays income to Jennifer's daughter for life, then distributes the remainder to the grandchild of a friend who is 31 years old.

A) A gift of a remainder interest in a trust just established that is paying an income interest to Jennifer. The remainderman is a grandson whose parents died in an auto accident earlier this year and before the inception of the trust. Statements (B), (C), and (D) all have skip persons as transferees and will be subject to GST. Only the grandson in statement (A) is not a skip person. His parent's death moved him up a generation. The 37-1/2 year rule does not apply to lineal descendants.

Elizabeth has drafted her own will using the "EZ Wills" software that she purchased on the internet. She sends it to you for a review. In your first review of the will, you look for which of the following common provisions? A) A statement of the domicile of the testator B) A secondary clause C) A specific bequest of property owned in tenancy by the entirety D) A disclosure clause

A) A statement of the domicile of the testator A statement of the domicile of the testator is a provision that is commonly found in a will and establishes the jurisdiction for the will to be administered. Neither a secondary clause nor a disclosure clause exists. Property owned in tenancy by the entirety transfers by operation of law and is not disposed of through a will.

Which of the following statements accurately reflects the nature of buy-sell agreements? A) A stock redemption plan must have a corporation as a party to the contractual arrangement. B) A stock redemption plan increases the cost basis of surviving shareholders. C) Under a cross-purchase plan funded with life insurance, premiums paid are tax deductible to the payor. D) Proceeds of a life insurance policy owned by a surviving shareholder must be included in the gross estate of the decedent.

A) A stock redemption plan must have a corporation as a party to the contractual arrangement. The corporation must be the purchaser the stock redemption plan. Statement (B) is incorrect because a stock redemption plan is a stock purchase by a corporation, so the cost basis of the surviving shareholders is not affected. Statement (C) is incorrect because premiums are not tax deductible. Statement (D) is incorrect because proceeds of a policy owned by a surviving shareholder are not includible in the decedent's gross estate.

Chad and Ryan have been partners for the past 25 years. They are not married. Chad's family is quite wealthy, and and is also very conservative. They do not approve of Chad's relationship with Ryan. Chad was diagnosed with cancer last year and was given only 12-15 months to live. Chad plans to leave his substantial wealth to Ryan. Over the holidays, Chad and his family argued about his estate plan. Chad has come to you, an estate planning attorney, and asks you to recommend ways he can ensure that Ryan will receive his assets. Which of the following would you be least likely to recommend to Chad to meet his objectives? A) A well-drafted will leaving everything to Ryan with a no-contest clause. B) A revocable living trust created and funded now with Ryan as the beneficiary at Chad's death. C) An irrevocable trust created and funded with Chad as the income beneficiary and Ryan as the remainder beneficiary. D) Retitling all assets as JTWROS with Ryan.

A) A well-drafted will leaving everything to Ryan with a no-contest clause. While all of these options may accomplish Chad's goal, statement (A) has the most inherent risk. The use of a will in this situation is very susceptible to a contest. The no-contest clause is irrelevant, because Chad did not leave anything to anyone else to encourage them not to contest. The trust and titling options are much less likely to be susceptible to fraud and undue influence claims.

Which of the following is NOT a terminable interest? A) An ownership interest in a life insurance policy. B) A life estate in a home. C) An interest in a patent. D) A term interest in property for 10 years.

A) An ownership interest in a life insurance policy. The ownership interest of a life insurance policy is not a terminable interest because the ownership interest does not terminate. All of the other interests listed are terminable interests. A life estate is a terminable interest, because the interest in the property terminates at the individual's death. An interest in a patent is a terminable interest, because a patent has the right to terminate after a certain period of time. Statement (C) describes an interest ending after 10 years, so it is also a terminable interest.

Anne recently died. Anne is survived by her husband, Edward, and daughter, Catherine. Which of the following would be a qualifying property transfer for the purposes of the unlimited marital deduction? A) Anne leaves ownership of certain copyrights to Edward. B) Property transferred to a credit shelter trust for the benefit of Catherine, with Edward as the trustee. C) Anne leaves her beach house to Edward, subject to the condition that if Edward does not survive Anne's sister, Anne's sister will get the property. D) The $1,000,000 life insurance policy on Anne's life that is owned by Edward.

A) Anne leaves ownership of certain copyrights to Edward. Although copyrights are terminable interests, no person other than Edward has any interest in the property, since all rights were given to Edward. Therefore, the transfer of the copyrights to Edward will qualify for the marital deduction. Statement (B) does not qualify for the unlimited marital deduction because even though Edward is trustee and has legal title to the property inside the trust, he does not have beneficial title to the property. Statement (C) does not qualify for the unlimited marital deduction because the transfer to Edward is a terminable interest. Statement (D) does not qualify for the unlimited marital deduction because the proceeds of a life insurance policy owned by Edward on Anne's life will not be included in Anne's gross estate.

Which of the following applies to the income tax or estate tax treatment of life insurance policy proceeds? A) Benefits received under a periodic settlement option are partially subject to income tax. B) Death proceeds are includible in the gross estate of the decedent if the decedent was the insured, regardless of ownership. C) Payments under a cashout settlement option are partially subject to income tax. D) For a personally owned life insurance policy, premiums are deductible if made as part of a court-ordered child or spousal support plan (QDRO).

A) Benefits received under a periodic settlement option are partially subject to income tax. Periodic annuity settlement benefits are partially subject to income tax, because the recipient has a tax basis only equal to the original proceeds. Statement (B) is incorrect because proceeds are includible in the estate for tax purposes only if grantor retained an incident of ownership. Statement (C) is incorrect because cashout options are not subject to income tax. Statement (D) is incorrect because life insurance premiums are not deductible when personally owned.

Trey decides to set up a trust for the benefit of his two sons, Ronnie and Chad. Trey makes an annual contribution to the trust in the amount of $30,000 and gives each son the right to withdraw up to $15,000. In the current year, when the total trust assets are $52,000, Ronnie decides to withdraw $15,000, but Chad does not withdraw anything. What is the result of Chad's decision not to withdraw any of Trey's contribution to the trust? A) Chad has made a taxable gift to Ronnie of $5,000. B) Ronnie has made a taxable gift to Chad of $15,000. C) Trey has made a taxable gift to Ronnie of $15,000. D) No taxable gifts are made by Chad.

A) Chad has made a taxable gift to Ronnie of $5,000. This question addresses the 5/5 Lapse Rule. The 5/5 Lapse Rule states that a taxable gift has been made where a power to withdraw in excess of $5,000 or 5% of the trust assets is lapsed by the powerholder. In this case, Chad has allowed his power to withdraw $15,000 to lapse. As a result, Chad has made a gift to himself of $5,000 ($7,500 - ($5,000/2)) and a gift to Ronnie of $5,000 ($7,500 - ($5,000/2)).

You are opening a new financial planning practice and you would like to put together a team of experts to help your clients with estate planning. Which of the following groups represents the best team to help your clients? A) Financial planner, CPA, and attorney. B) CPA, psychiatrist, and insurance salesman. C) Financial planner, attorney, and real estate agent. D) Attorney, insurance salesman, and IRS agent.

A) Financial planner, CPA, and attorney. The best team for your client would include a financial planner, CPA, and attorney. A licensed insurance specialist and real estate agent are also good additions to an estate planning team, but the team described in statement (A) covers all of the required aspects of an estate planning team: financial planning, tax planning, and legal representation.

Which of the following retained interests of a grantor to an irrevocable trust will cause inclusion to the grantor's gross estate? A) Having the ability to direct the amount that a remainder beneficiary will receive by retaining the right to the amend the trust document. B) The ability to assist a corporate trustee in an investing the trust assets to achieve the trust's objective. C) The ability to add to the trust to help the trustee purchase a new life insurance policy on the grantor and her husband, where the trust is the applicant, owner and beneficiary. D) Having given up the ability to receive an annuity payment from the trust 5 years prior to their death.

A) Having the ability to direct the amount that a remainder beneficiary will receive by retaining the right to the amend the trust document. (B), (C), will not cause inclusion. Answer (D) would have cased inclusion had the grantor not ceased the payments more than 3 years prior to her death. (A) will cause inclusion in the grantor's gross estate as the grantor has retained an income interest.

Which type of will is handwritten and does not generally require a witness? A) Holographic B) Oral C) Nuncupative D) Statutory

A) Holographic Holographic wills are handwritten. The material provisions of the will must be in the testator's handwriting. The will must be dated and signed by the testator, and does not generally need to be witnessed.

Tracey is a financial planner who recently received his CFP® designation. Tracey does not have any other designations or licenses. Although Tracey's expertise is investment planning, he is anxious to expand his client base and is willing to assist clients with any area of financial planning. Over the last month, Tracey engaged in the following activities with Troy, a new client. I. During the initial meeting, Tracey collected personal data about Troy, including the estate planning documents Troy had previously executed. II. During the second meeting, Tracey recommended the use of a trust to fulfill some of Troy's estate planning goals. III. Troy called Tracey one afternoon and asked if Tracey could explain the probate process to him, which Tracey promptly did. IV. Tracey downloaded a copy of a generic will from the internet, filled in Troy's information and gave the document to Troy to be executed. Of the activities above, which would be considered the unauthorized practice of law? A) IV only B) II and III C) III and IV D) II, III, and IV E) I, II, III, and IV

A) IV only Only statement four would be considered the unauthorized practice of law. The drafting of legal documents is reserved for attorneys. Inquiring about estate planning documents should be completed by all practitioners. Recommending appropriate estate planning devices, such as trusts, can be done by financial planners. Explaining the probate process to a client would not be the unauthorized practice of law; the line would be crossed if Tracey gave legal advice regarding the probate process.

Which of the following is true regarding GRATs? A) If the grantor dies during the trust term, the assets will be included in her estate. B) The grantor may revoke the assets in a GRAT at any time. C) The transfer of assets into a GRAT is an incomplete gift until the end of the trust term. D) GRATs are testamentary trusts.

A) If the grantor dies during the trust term, the assets will be included in her estate. Statement (B) is incorrect because the transfer is irrevocable. Statement (C) is incorrect because the transfer is irrevocable, thereby making it a completed gift at the time of the transfer. Statement (D) is incorrect because GRATs are created during the grantor's lifetime, not a testamentary trust (created in a decedent's will).

Jonathan worked with an estate planner for several years prior to his death. Accordingly, Jonathan made many transfers during his life in an attempt to reduce his potential estate tax burden. Jonathan's executor, Timothy, is thoroughly confused. Timothy comes to you for clarification on which assets to include in Jonathan's gross estate. All the following transactions will be included in Jonathan's gross estate EXCEPT: A) Jonathan gave $40,000 to each of his three grandchildren 2 years ago. No gift tax was due on the gifts. B) Jonathan purchased a life insurance policy on his life. The policy has a face value of $300,000. Jonathan transferred the policy to his son 2 years ago. C) Jonathan and his wife owned their personal residence valued at $250,000 as tenants by the entirety. D) After inheriting a mountain vacation home from his mother, Jonathan gifted the vacation home to his daughter to remove it from his gross estate. Jonathan continued to use the property as a weekend getaway and continued all maintenance on the property.

A) Jonathan gave $40,000 to each of his three grandchildren 2 years ago. No gift tax was due on the gifts. The $40,000 gifts to his grandchildren are excluded from his gross estate because only gifts of life insurance within 3 years of the transferor's death and any gift tax paid on such a gift are included in a transferor's gross estate. The life insurance policy in option (B) is included in Jonathan's gross estate because transfers of life insurance within 3 years of death are included in the decedent's gross estate. Any property owned at the decedent's date of death, as in option (C), is included in the decedent's gross estate. (Do not confuse gross estate inclusion with probate inclusion.) Even though, in option (D), the value of the mountain home Jonathan gave to his daughter generally would not be included in Jonathan's gross estate under other circumstances, the fact that Jonathan continued to use the property each weekend and maintained the property would cause inclusion in his gross estate.

Chris and Jenn gave their son, Evan, a car worth $4,000. The car was originally owned by Chris and Jenn as community property. Evan is married to Michelle and lives in a community property state. After the gift, how is Evan's ownership of the car classified? A) Sole Ownership B) Joint tenants with Chris and Jenn C) Tenancy in Common with Michelle D) Community Property with Michelle

A) Sole Ownership The car is owned by Chris as sole owner. Statement (B) is incorrect because there is no indication that Chris or Jenn retained any interest in the car after the gift. Statement (C) is incorrect because the gift was made only to Evan. Statement (D) is incorrect because even though Evan is married, a gift to an individual would not be community property.

Which of the following individuals died testate? A) Manuel met with his attorney and prepared a will, leaving all of his assets to charity shortly before his death from a heart attack. B) Paula wrote an email to her children before her death, designating which children would receive specific property when she died. C) Frank notified his attorney before his death in writing that he was satisfied with all of his property going to his wife and children, so he had no need for a will. D) Kayla, a stay at home mother, did not prepare a will before she died because she owned no assets in her name.

A) Manuel met with his attorney and prepared a will, leaving all of his assets to charity shortly before his death from a heart attack. Even though Manual left all of his assets to charity, his will was valid, so he died testate. The other answers describe individuals who died intestate (without a valid will). Statement (B) is incorrect because an email is not satisfactory as a valid will. Statements (C) and (D) are not correct because even though in both cases the individuals did not believe they needed a will, they failed to create one before their deaths.

Which of the following is not a common estate planning goal? A) Maximizing transfer costs. B) Minimizing transfer taxes. C) Providing for liquidity at death. D) Fulfilling client's healthcare decisions.

A) Maximizing transfer costs. Maximizing transfer costs is not an estate planning goal. Transfer costs should be minimized to the greatest extent possible, while also fulfilling the client's goals. All of the other answers are common estate planning goals.

Which of the following is included in the gross estate? A) Proceeds from a life insurance policy owned by the insured decedent that were assigned to an ILIT two years before death of the insured. B) A secular trust where the only income beneficiary was the decedent's spouse. C) Property where the decedent had a reversionary interest of less than 1% of the value. D) Gift taxes paid two years prior to the decedent's date of death for gifts made four years earlier.

A) Proceeds from a life insurance policy owned by the insured decedent that were assigned to an ILIT two years before death of the insured. Incidents of ownership of life insurance policies assigned within three years of death are includible in the decedent's estate as a gift, as are CRATs and CRUTs. Any amount of gift tax subject to the gross up rule is includible in the taxable estate but must be for gifts made within three years of death.

The executor of an estate liquidated assets to generate the cash necessary to pay the estate taxes. Of the following assets, which is the least likely to generate income tax consequences upon its sale? A) Real estate sold within three months of the decedent's date of death. B) Publicly traded securities held for 8 years, sold two weeks after the decedent's date of death. C) The redemption of the stock of a closely held business. The redemption did not qualify for Section 303 treatment. D) The decedent's antique and rare car sold to an automobile collector.

A) Real estate sold within three months of the decedent's date of death. The real estate sold within three months of the decedent's date of death would not generally create any income tax consequences, because the fair market value on the estate tax return of that piece of real estate would be that sales price. So when the estate sold the real estate, it would not have any gain or loss on the transaction, because its adjusted basis (the fair market value on the estate tax return) would be equal to the proceeds of the sale. Statement (B) would create income tax consequences, as the adjusted basis of the securities to the estate would be the fair market value of the securities at the decedent's date of death. Since these are publicly traded securities, their value changes daily, and the estate would most likely have some gain or loss on the sales. The stock redemption in statement (C) would create tax consequences because non-303 redemptions are treated as dividend distributions, and are therefore subject to income tax. Statement (D) would create income tax because the gain on sale of property is taxable income.

Which of the following is an appropriate means of valuing an asset in an estate? A) Real estate valued by an appraiser. B) Closely held business interests can be valued per the public stock exchange. C) Life insurance policies owned by the decedent on the life of another are valued at zero if the insured has not yet passed. D) Discounts can be applied to closely held business interests for majority ownership and preferential voting rights.

A) Real estate valued by an appraiser. Real estate is a unique asset and should be valued by an appraiser. Statement (B) is incorrect because closely held businesses are not traded on the public stock exchange. Statement (C) is incorrect because life insurance policies owned by the decedent are included in the decedent's gross estate. Statement (D) is incorrect because these rights would add a premium to the value, not a discount.

Robert has a falling out with his adult children and wants to "write them out of his will." Which of the following is an effective means of fulfilling Robert's wishes? A) Robert throws his old will into a fire and writes a new will leaving everything to charity. B) Robert sends his children a certified letter telling them they have been disinherited. C) Robert directs his attorney to call each of his children to tell them they have been disinherited. D) Robert calls his attorney and tells her that he wants to disinherit his children.

A) Robert throws his old will into a fire and writes a new will leaving everything to charity. A will can be revoked by physically destroying the will. None of the other answers would effectively revoke Robert's will.

Your client, Samantha, is 65 years old and she is interested in establishing a trust with a value of $10,000,000 for her family. She is aware of the Generation Skipping Transfer (GST) Tax, and has asked you for your advice as to which of the following would not be considered a skip person. Who would not be considered a skip person for GST Tax purposes? A) Samantha's son, Sean, who is 35. B) Samantha's granddaughter, Mia, who is 10 C) Samantha's grandson, Max, who is 12 D) A trust that Samantha had established 2 years ago for her favorite employee, Jill, who just turned 25.

A) Samantha's son, Sean, who is 35. Due to the age difference of more than 37½ years and the non-related party status, the trust for Jill is a skip person. The reason Sean is not a skip person is because he is a first-generation descendant. Mia and max are skip persons because they are two generations younger than the transferor.

Reina and her husband William gave Reina's nephew, Burt, a car worth $7,000 that they owned as community property. After the gift, how is the nephew's ownership of the car classified? A) Sole Ownership B) Community Property with Burt's wife C) Tenancy in Common with Reina and William D) Joint Tenancy with Right of Survivorship

A) Sole Ownership The car is owned by the nephew as the sole owner because Reina and William did not retain any ownership in the car. Gifted property is considered separate property and would not be owned as community property with Burt's wife.

Alton would like to transfer the ownership of his Picasso painting to his son Edgar, but Alton would like to continue to have the painting hang in his house for the next five years. Which of the following would you recommend to Alton? A) TPPT B) CRAT C) QPRT D) FLP

A) TPPT Tangible Personal Property Trusts will transfer ownership of personal property while allowing Alton use of the painting for a period of time. Statement (B) is incorrect because Alton's son Edgar is not a charity. Statement (C) is incorrect because a QPRT is for the transfer of a personal residence. Statement (D) is incorrect because an FLP would be more appropriate for transferring ownership of a family business asset.

Holden is the executor of his mother's estate. Holden determines that his mother's gross estate contains the following assets and debts: $450,000 of real estate $300,000 in cash and cash equivalents $670,000 in qualified retirement plans $370,000 outstanding debt Prior to her death, his mother's home caught fire and the damage totaled $26,000. Administrative fees to manage the estate total $136,000. What is the adjusted gross estate? A) $1,420,000 B) $914,000 C) $888,000 D) $244,000

B) $914,000 The adjusted gross estate = gross estate - administrative/burial/debt/casualty losses after death. The Gross Estate = $1,420,000 ($450,000 real estate + $670,000 retirement + $300,000 cash). Deductible losses = $506,000 ($370,000 debt + $136,000). $1,420,000 - $506,000 = $914,000.

A trust that is set up to benefit a child under the age of 21 that must terminate and distribute all assets to the child at the age of majority describes which type of trust? A) 2503(b) Trust. B) 2503(c) Trust. C) UGMA Account. D) UTMA Account.

B) 2503(c) Trust. A 2503(c) trust allows income to be accumulated within the trust until the minor beneficiary attains the age of majority. The transfer of property to the trust qualifies for the annual exclusion. A 2503 (b) trust has to distribute assets annually on behalf of the child yet can hold the assets in trust beyond the age of majority. Answers (C) and (D) are custodial accounts and not trusts.

Nate owns the following property: • A personal residence titled as fee simple valued at $500,000. • A $500,000 life insurance policy on his own life. The only named beneficiary is Nate's brother Jaime, who died 6 months ago. • A car valued at $15,000 titled as JTWROS with Nate's mother. • An IRA valued at $400,000 with Nate's mother as the named beneficiary. What is the current value of Nate's probate estate? A) $500,000. B) $1,000,000. C) $1,400,000. D) $1,415,000.

B) $1,000,000. The probate estate will include the personal residence and the life insurance policy. Life insurance policies are usually not included in probate, but in this instance, the beneficiary named is deceased and is therefore not available to take the proceeds. Therefore, the death benefit must be included in the probate estate so the court can determine how it will be distributed. The car is not included because of the JTWROS ownership, and thus it transfers by operation of law. The IRA is not included because there is a living named beneficiary and thus will transfer via contract law.

Perry's father sold the family business to him using a private annuity. The private annuity was structured such that Perry would pay his father $40,000 per year plus interest, for the remainder of his father's life. At the date of the sale, Perry's father's life expectancy was 20 years and Perry's father is in great health. After six years, Perry's father died of a heart attack and Perry sold the business for $2,000,000 six months after his father's death. What is Perry's capital gain/loss on the transaction? A) $240,000 B) $1,760,000 C) $1,200,000 D) $2,000,000

B) $1,760,000 A buyer's adjusted basis of property purchased with a private annuity is equal to the sum of all annuity payments paid. In this scenario, Perry made six annuity payments of $40,000, or a total of $240,000. Since he sold the property for $2,000,000, his gain is calculated by subtracting his basis from the sales price to arrive at $1,760,000 ($2,000,000 - $240,000).

Kathi and Darrin, who are married, live in a community property state. They purchased the home 17 years ago for $100,000 using joint funds. After many improvements and a surge in the market, the home is now worth $200,000. If Darrin died today and left his share of the home to his daughter Elizabeth, what is Kathi's basis in the home? A) $50,000. B) $100,000. C) $150,000. D) $200,000.

B) $100,000. Kathi's one-half interest in the home will have a basis of $100,000 due to a step-to fair market value of both halves at Darrin's death because the property is owned as community property. Their daughter will receive her half of the home with step up basis as well.

Mimi and John are married and have always lived in a community property state. They purchased the home 25 years ago for $140,000. After many improvements and a surge in the market, the home is now worth $500,000. John and Mimi both have wills leaving their entire estates to their daughter, Bette. What would Mimi's basis in the home be if John died today? A) $500,000. B) $250,000. C) $70,000. D) $320,000.

B) $250,000. Mimi's one-half interest in the home will have a basis of $250,000 due to a step-to fair market value of both halves at Johns' death because the property is owned as community property.

Jane and John recently sold some land they owned for $500,000 this year. They received the land and a check equal to the amount of the then-current annual exclusion five years ago as a wedding gift from Brody's Aunt Jeanette. Aunt Jeanette purchased the land many years ago when the property was worth $200,000. At the date of the gift, the property was worth $300,000 and Aunt Jeanette paid $65,000 in gift tax. What is the long-term capital gain on the sale of the property? A) $200,000 B) $278,333 C) $300,000 D) $500,000

B) $278,333 In general, when a donor makes a gift of property the donee will take the property at the donor's adjusted basis. The holding period of the donee will include the holding period of the donor for purposes of subsequent transfers and the determination of long- or short-term capital gains. An exception to the general basis rule occurs when the donor gives property with a fair market value in excess of his adjusted basis and the donor pays gift tax. The gift tax associated with the appreciation is added to the donor's original adjusted basis to determine the donee's basis. Thus, the basis would be: $200,000 (OB) + ($65,000 (GT) x $100,000/$300,000) = $$221,667 The gain on the asset would be: $500,000 - $221,667 = $278,333.

Drake was a majority owner in a closely held business. He had an adjusted basis in his interest of $600,000, and at his death this year, the fair market value reported on his estate tax return was $5,000,000. Like most majority owners in closely held businesses, Drake did not have much liquidity in his estate and his executor was forced to redeem some of his interest in the business. If Drake's executor redeemed 30 percent of Drake's interest for $1,800,000 to pay the estate tax and administration fees, how much is subject to capital gains tax? A) $0. B) $300,000. C) $1,200,000. D) $1,800,000.

B) $300,000. Drake's estate would have an adjusted basis in the 30 percent interest equal to 30 percent of the fair market value at Drake's date of death, or $1,500,000. If the executor of Drake's estate sold the interest for $1,800,000, the gain of $300,000 ($1,800,000-$1,500,000) would be subject to capital gains tax under Section 303 (only available at the death of the owner). Ordinarily, unless a redemption is a complete redemption, the redemption is treated as a dividend.

Bob, knowing that he and his wife will most likely be assessed transfer taxes at their death, called his attorney Mary and suggested that he would like to set up a device by which he can retain an income stream, as he still needs the payout, yet have the assets go to his kids while he is still alive. Mary suggested that Bob and his wife fund a Grantor Retained Annuity Trust (GRAT) with $1,000,000 from his brokerage account that is managed by his advisor Linda. Linda will manage the assets so as to pay the agreed upon grantor retained annuity. In the terms of the trust, Bob is to receive an annuity payment of $40,000 for 10 years. Assuming Bob is the sole grantor, what is the gift that Bob has made to the trust? A) $400,000 B) $600,000 C) $1,000,000 D) $2,000,000

B) $600,000 In valuing gifts to grantor retained trusts, use the subtraction method: Gift value = FMV of the property transferred - the value of the retained interest. $1,000,000 - $400,000 = $600,000.

Watson owns the following property: A solely owned home worth $500,000. His basis in the home is $380,000. A $200,000 life insurance policy on his own life. His sister, who died last year, is the only named beneficiary. A boat valued at $25,000 titled as JTRWROS with his son. What is the current value of Watson's probate estate? A) $500,000. B) $700,000. C) $725,000. D) $525,000.

B) $700,000. The probate estate will include the personal residence and the life insurance policy. The life insurance policy is included because the named beneficiary was already deceased at the time of Watson's death, therefore, the property will be classified as probate and distributed in accordance with the state's intestacy laws. The boat is not included because JTWROS property is non-probate property that transfers by operation of law.

Donna and Daniel have lived in Louisiana their entire marriage. Currently, their combined net worth is $4,000,000 and all of their assets are community property. After meeting with their financial advisor, Donna and Daniel begin a plan of lifetime gifting to reduce their gross estates. During 2020, they made the following cash gifts: Son $80,000 Daughter $160,000 Republican National Committee $75,000 Granddaughter $15,000 What is the amount of the taxable gifts to be reported by Donna? A) $35,000. B) $90,000. C) $127,500. D) $255,000.

B) $90,000. Because the assets are community property, the gifts are deemed to be made 50% by each spouse. Gift-splitting is not an issue. The cash payment to the Republican National Committee is not a gift for gift tax purposes

Which of the following is correct regarding a power of attorney? A) The principal is the individual that is granted decision-making power. B) A guardian ad litem is an agent of the principal. C) The agent is the person granting the power. D) The attorney who prepares the document is the power of attorney.

B) A guardian ad litem is an agent of the principal. The parties to a power of attorney are the principal and the agent. A guardian ad litem is an agent acting on behalf of the principal for a specific purpose (usually a minor child in this instance). Statement (A) is incorrect because the principal is the individual who is granting the power to the agent to make decisions or act on their behalf. Statement (C) is incorrect because the principal is the person granting the power. Statement (D) is incorrect because the attorney is simply serving an administrative function to execute the document.

Which of the following statements is incorrect regarding wills? A) A holographic will is a handwritten will. B) A nuncupative will is a will that has no witnesses. C) A statutory will is a will created according to the state's laws. D) A joint will is one will for two testators who leave their property to each other.

B) A nuncupative will is a will that has no witnesses. An oral/nuncupative will is an oral dying declaration made before sufficient witnesses per state law. All of the other statements are true.

Which of the following statements relating to qualified transfers for gift tax purposes is not correct? A) The relationship between the donor and the donee is irrelevant with a qualified transfer. B) A payment made directly to an individual for the purpose of paying medical expenses is a qualified transfer. C) The exclusion for a qualified transfer may exceed the annual exclusion. D) A payment made to a qualified education institution for tuition costs is a qualified transfer.

B) A payment made directly to an individual for the purpose of paying medical expenses is a qualified transfer. A payment made directly to an individual to reimburse him for medical expenses is not a qualified transfer. To be a qualified transfer, the payment must be made directly to the healthcare provider. All of the other options are true.

Which of the following accurately describes a life estate? A) An interest in property for a specified number of years. B) An interest in property that ceases upon the death of an individual. C) An undivided interest in property held by two or more related or unrelated persons. D) A complete interest in property with all the rights associated with outright ownership.

B) An interest in property that ceases upon the death of an individual Statement (B) is the definition of a life estate. Statement (A) is the definition of an interest for a term. Statement (C) is the definition of tenancy in common. Statement (D) is the definition of fee simple.

Maria owns a rental property. She would like to jointly own the property with her husband, Miguel and leave the property to her daughter, Sophia. Which of the following ways could she title the property? A) As joint tenants with right of survivorship with Miguel. B) As tenants in common with Miguel. C) As community property, regardless of which state they live in. D) As tenants by the entirety so she can sell the property without Miguel's consent if she needs to.

B) As tenants in common with Miguel. Tenants in common may bequest property as they wish. Statement (A) is incorrect because JTWROS passes directly to the surviving tenants, and cannot be passed to anyone else. Statement (C) is incorrect because only community property states allow community titling. Statement (D) is incorrect because tenants by the entirety need the other tenants' consent to sell the property.

Which of the following is an advantage of a revocable living trust? A) Reduction in federal estate taxes B) Avoidance of probate C) Removal of asset appreciation from the grantor's gross estate D) Distribution of the trust assets according to the grantor's will's terms

B) Avoidance of probate Assets in a revocable living trust transfer by contract and outside of probate. Statements (A) and (C) are incorrect because use of a revocable living trust does not reduce the grantor's federal estate taxes, because the full fair market value of the trust assets is included in the grantor's gross estate. Statement (D) is incorrect because the trust agreement, not the grantor's will, controls the distribution of the trust assets.

BE Six months ago, Jeanette transferred a piece of real estate to her son, Charles. Jeanette purchased the real estate for $90,000 6 years ago, and the property was valued at $65,000 on the date of transfer. Jeanette paid $20,000 in gift tax on the transfer. All the following statements are true EXCEPT: A) If Charles were to sell the property for $60,000 today, the loss would be a short-term capital loss. B) Charles's basis will be $65,000. C) Charles will have a dual basis for income tax purposes. D) If Charles sold the property for $120,000 after holding it for 5 years, his gain would be $30,000.

B) Charles's basis will be $65,000. Because Jeanette's basis in the property was greater than the fair market value (FMV) of the property on the date she gifted it, Charles will be subject to the double basis rules and will receive no adjustment in basis for gift tax paid.

Hazel, a widow, died. She had made no previous lifetime taxable gifts and she died with a gross estate of $11,200,000, consisting solely of a diversified portfolio of publicly traded, income-producing stocks. Her debts were $75,000 and estate administrative expenses that amounted to $50,000. Which of the following post-mortem techniques should Hazel's executor consider electing? A) Elect the alternative valuation date. B) Deduct estate administrative expenses on the estate's fiduciary income tax return. C) Pay estate taxes under IRC Section 6166. D) Use a Section 303 stock redemption.

B) Deduct estate administrative expenses on the estate's fiduciary income tax return. There is no estate tax due, therefore the alternative valuation and installment payment option is not needed. The estate is not a closely held business (C corporation), so Section 303 redemption does not apply.

Which of the following is true regarding taxable gain on the sale of inherited property for income tax purposes? A) The gain on the sale of inherited property is classified as long- or short-term capital gain depending on how long the beneficiary held the property before the sale. B) The beneficiary inherits the property at step-up basis. C) The beneficiary inherits the property at carryover basis. D) The beneficiary must pay estate tax on the property inherited.

B) The beneficiary inherits the property at step-up basis. Beneficiaries/legatees inherit property with step-up basis. This is the reason statement (C) is incorrect. Statement (A) is incorrect because the holding period for inheritances is long term regardless of how long the decedent or the legatee held the property. Statement (D) is incorrect because the beneficiary is not liable for paying estate tax.

Harriet and Wendy transfer $90,000 to an irrevocable trust, naming their three children, Renaldo, Francisco and Bartolo as the beneficiaries. The trust provisions include a right to withdraw an amount equal to 1/3rd of any contribution for 30 days for each beneficiary up to the annual exclusion limit for the 2 spouses ($30,000 for 2021). After 30 days, all the brothers have lapsed the power to withdraw the $30,000 available to them. Which statement is true regarding the brother's lapsing of their powers? A) Each has made a taxable gift to the others Renaldo and Bartolo in the amount of $5,000. B) Each has made a taxable gift to the others of $8,333. C) Each has made a taxable gift to the others of $15,000. D) All of the above.

B) Each has made a taxable gift to the others of $8,333. This question addresses the 5/5 Lapse Rule. Which states that a taxable gift has been made where a power to withdraw in excess of $5,000 or 5% of the trust assets is lapsed by the powerholder. In this case, all 3 brothers have allowed their power to withdraw $30,000 to lapse. As a result, all 3 have made gifts to themselves and to their brothers of $8,333 ($30,000 - $5,000= $25,000/3)

Which form is used to report a testamentary transfer subject to GSTT? A) Form 1040 B) Form 706 C) Form 1041 D) Form 709

B) Form 706 Testamentary transfers subject to GSTT are reported on the Form 706, the United States Estate and Generation-Skipping Transfer Tax Return form.

Mary Jane's husband died in October of 2021. Mary Jane has a one-year-old dependent child and has not remarried. Which filing status will Mary Jane use on her 2024 income tax return? A) Married filing separately B) Head of household C) Married filing jointly D) Qualifying widow

B) Head of household Mary Jane will file as head of household in 2024. Since Mary Jane's husband died in 2021, she will file married filing jointly for the year of her husband's death. In the two years after her husband's death (2022 and 2023), Mary Jane will file as qualifying widow. For the 2023 tax year, Mary Jane will file head of household, and this will continue until she remarries or no longer provides a home for her child.

Of the following, who should generally be a member of the estate planning team? I. Investment Advisor II. Trustee III. Insurance agent IV. Attorney A) I and II B) I, III, and IV C) I, II, and III D) I, II, III, and IV

B) I, III, and IV A trustee is not a member of the estate planning team, because the trustee's responsibility is not to advise the client about forming the estate plan but to carry out the terms of the trust document.

Assume the decedent did not have a will. Which of the following property would be included in the decedent's probate estate? 1.) A Pay on Death checking account owned by the decedent that will be transferred to the decedent's grandchild. 2.) Community interest property owned with the decedent's wife 3.) Retirement plan proceeds made payable to the decedent's daughter 4.) A mountain vacation home the decedent owns jointly (JTWROS) with his siblings A) I and II B) II only C) I, II and IV D) None of the above

B) II only Options I, III and IV are not included in the probate estate. POD and retirement contracts are transfers by contract law, and property owned JTWROS transfers by operation of law to the surviving joint tenants.

Jack and his wife, Carol, were in an auto accident. Carol died immediately, and Jack died of complications three weeks later. His gross estate was $12.4 million. One of the major assets in his estate was closely held stock in a rapidly appreciating business. His estate had unsecured debts of $400,000 and administrative expenses of $75,000. His will allocates his estate to his children in equal shares. Which post-mortem planning techniques might benefit Jack's estate? I. The alternative valuation date II. A Section 303 stock redemption III. The QTIP election IV. Special use valuation V. Installment payment of estate taxes A) III only B) II, IV, and V C) IV and V D) I, III, and IV

B) II, IV, and V A section 303 redemption would be effective because Jack's estate will need sufficient liquidity to pay estate taxes and costs. Special use valuation will likely be useful to lower the overall business value and tax amount. Installment payments will be useful to spread the tax liability over time and lessen the immediate liquidity pressures on the estate. Due to a rapid increase in asset value, statement I would likely provide a higher estate value, and therefore, the alternate valuation date is not likely useful. Statement III is not useful because the QTIP election is not available since Carol died first and no assets pass to her under Jack's will.

Three siblings want to start a bed and breakfast. The two older siblings have stable incomes and minimal debt. The youngest sibling has not been consistently employed and she has significant debt from her credit cards and student loans. The two older siblings can contribute most of the purchase price, while the youngest will contribute less but will take on most of the management of the property. Which of the following is correct regarding titling of the property? A) If they title the property as tenants in common, each sibling will have a survivorship right to a deceased sibling's ownership interest. B) If they title the property jointly with right of survivorship, creditors of the third sibling can potentially claim the property of the other two siblings to satisfy the third siblings' debt. C) If they title the property jointly with right of survivorship, they can allocate ownership percentages according to the amounts they contribute. D) If they title the property as tenants in common, creditors of the third sibling can potentially claim the property of the other two siblings to satisfy the third siblings' debt.

B) If they title the property jointly with right of survivorship, creditors of the third sibling can potentially claim the property of the other two siblings to satisfy the third siblings' debt. The property owned by joint tenants with right of survivorship could potentially be claimed by creditors of one of the tenants. Co-owners of tenancy-in-common property are not liable for the debts of the other co-owners.

Which of the following is true regarding SCINs and private annuities? A) Collateral may be taken against assets in both a private annuity and a SCIN. B) In both types of agreements, no payments after the date of death are included in the seller's gross estate. C) In both types of agreements, the payment is determined by an IRS calculation. D) In both types of agreements, the buyer's basis is the sum of the payments made.

B) In both types of agreements, no payments after the date of death are included in the seller's gross estate. Statement (A) is incorrect because a collateral interest can only be used in a SCIN. Statement (C) is incorrect because in a SCIN, the payments are determined by the buyer and seller. The IRS calculation is used to determine payments for an annuitant. Statement (D) is incorrect because in a SCIN, the buyer's basis is the sale price. Even if the seller dies before the end of the SCIN term, the capital gain is considered an estate transfer to the buyer. Therefore, the buyer's basis will be the sale price.

Why is a springing durable power of attorney useful? A) In the event that the principal becomes disabled, the power of attorney will automatically become ineffective. B) It allows the principal to choose an agent to make decisions on her behalf if she becomes unable to do so. C) It ensures the decedent's property will be distributed according to her wishes. D) It provides medical professionals with detailed instructions on the disabled individual's wishes for medical care.

B) It allows the principal to choose an agent to make decisions on her behalf if she becomes unable to do so. A springing durable power of attorney becomes effective only when the principal becomes disabled, and lasts as long as the disability. Statement (A) is incorrect because the durable nature of the power of attorney ensures that it remains valid through the principal's disability. Statement (C) is incorrect because this is referring to a will; the power of attorney can make decisions regarding the decedent's property that may or may not align with the decedent's wishes. Statement (D) is incorrect because this is referring to a living will.

Which of the following transfers would not be considered a qualified transfer? A) Kelsey pays $45,000 to Boston University for her niece's tuition. B) Kelsey pays $54,000 to her friend Emma, who uses the money to pay for her medical expenses. C) Kelsey pays $12,000 to Municipal Hospital for her granddaughter's medical expenses. D) Kelsey pays $14,000 to Elite Preparatory School for her nephew's tuition.

B) Kelsey pays $54,000 to her friend Emma, who uses the money to pay for her medical expenses. Options (A), (C), and (D) described qualified transfers. Option (B) is not a qualified transfer because the payment was not made directly to the health care provider.

Margie has come to you and said that she is considering executing a power of attorney for health care or an advance medical directive (also known as a living will). Although her state utilizes both documents, she believes that she only needs one of these documents. Which of the following statements is true regarding the two documents? A) Margie is correct in believing that an individual does not need both documents, she only needs to execute one document because they both accomplish the same goals. B) Margie should execute both documents as they cover different aspects of medical care. C) Margie only needs to execute the power of attorney for health care, because it covers everything the advance medical directive covers and more. D) Margie does not need to execute either document; she can solve her medical concerns by executing a DNR.

B) Margie should execute both documents as they cover different aspects of medical care. The documents address different medical care concerns. A power of attorney authorizes an agent to make decisions about her medical care, but generally does not address the ending of life-sustaining treatment. The living will addresses the ending of life-sustaining treatment, but not the providing of medical care. A DNR is not a replacement for the other two documents; it is an additional document that addresses the prevention of resuscitation in the event of heart failure for a terminally ill patient.

Harry owns a life insurance policy on his own life with a face value of $1,200,000. Annual premiums of $3,000 are paid on July 1 each year. He gifts the policy to his friend Megan on January 1 of this year. The balance of the interpolated terminal reserve was $12,000 on July 1 of the previous year and $14,000 on July 1 of this year. Which of the following is correct? A) Upon Harry's death, he made a taxable gift of $14,500 to Megan. B) On July 1, Harry made a taxable gift of $14,500 to Megan. C) On July 1, Harry made a taxable gift of $1,200,000 to Megan. D) If Harry dies next year, the death benefit will be excluded from his gross estate.

B) On July 1, Harry made a taxable gift of $14,500 to Megan. The value of life insurance for gift purposes is assessed at the time the policy is transferred and is calculated as the unearned premium plus the interpolated terminal reserve at the time of the gift. Since the policy was gifted mid-way through the policy year, half of the premium is gifted ($1,500) plus the mid-point of the balance of the interpolated terminal reserve ($13,000). Therefore, the gift value is $14,500. Option (A) is incorrect because the gift tax is assessed at the time of the transfer, not death. Option (C) is incorrect because the gift tax is assessed at the time of transfer and the gift tax value is not the death benefit. Option (D) is incorrect because if Harry dies next year, the death benefit will be included in his estate due to the 3-year rule.

Which of the following apply to Section 303 redemptions? A) The estate qualifies for a 303 redemption if it can show reasonable cause. B) Qualifying redemption amounts are limited by the payment of death taxes and estate administration taxes and costs. C) A publicly traded stock will usually qualify for a 303 redemption if the interest is a minority interest. D) The stock redeemed must be common stock.

B) Qualifying redemption amounts are limited by the payment of death taxes and estate administration taxes and costs. Statement (A) is incorrect because the estate only needs to show that it meets the 35% rule. Statement (C) is incorrect because 303 redemptions only apply to closely held stock. Statement (D) is incorrect because the stock can be either common or preferred.

Within the stage of the estate planning process which involves establishing the client relationship, what should be the last step? A) Asking the client for names of people similarly situated to the client. B) Sending an engagement letter detailing items discussed. C) Meeting with the client. D) Detailing services that may be provided.

B) Sending an engagement letter detailing items discussed. The chronological steps for this stage of the planning process include: 1.) Meeting with the client 2.) Detailing services that may be provided 3.) Sending an engagement letter to the client detailing items discussed in the meeting

A transfer to which of the following organizations would NOT qualify for the gift and estate unlimited charitable deduction? A) The Free Library Organization B) The Los Angeles Homeowner's Association C) Mothers Against Drunk Driving D) The Society for the Protection of Wild Birds

B) The Los Angeles Homeowner's Association Transfers to a homeowner's association do not qualify for the unlimited charitable deduction. Transfers to any of the other organizations would qualify for the unlimited charitable deduction.

Which of the following property interests qualifies for the unlimited marital deduction? A) John dies and leaves his vacation home to his wife as trustee of a testamentary trust that is created for the sole benefit of his two children. B) The executor of John's estate made the QTIP election for the bequest of a life estate interest in his personal residence to Deborah, John's wife. C) John bequeaths his interest in community property to his wife and subject to his wife surviving him by more than eight months. D) At John's death, his will created a trust for the benefit of his wife. The trust document gives his wife the authority to appoint assets to herself, her creditors, and her heirs with the approval of John's brother, Colin.

B) The executor of John's estate made the QTIP election for the bequest of a life estate interest in his personal residence to Deborah, John's wife. Only the property interest detailed in statement (B) would qualify for the marital deduction. Even though a life estate is a terminable interest, which is normally not eligible for the marital deduction, the QTIP election on the property qualifies the transfer for the marital deduction. Statement (A) does not qualify for the unlimited marital deduction, because the property does not transfer to the surviving spouse. She only holds an interest as trustee for John's children. Statement (C) does not qualify for the unlimited marital deduction because the survivorship clause cannot require the spouse to survive the decedent by more than six months. Statement (D) does not qualify for the unlimited marital deduction, because the surviving spouse cannot act alone to exercise the general power of appointment. To qualify, the surviving spouse must be able to exercise the general power of appointment alone.

Which of the following statements concerning the use of Crummey powers with an irrevocable life insurance trust is correct? A) Crummey powers are used to make trust contributions a future-interest gift. B) The right to exercise the Crummey powers must exist for a reasonable period of time each year. C) If there are several beneficiaries, Crummey powers should be given to only one beneficiary. D) A danger with the Crummey power is that the beneficiary can request an amount greater than the grantor's annual addition to the corpus.

B) The right to exercise the Crummey powers must exist for a reasonable period of time each year. Statement (A) is incorrect because Crummey powers are used to make an irrevocable life insurance trust a present-interest gift. Statement (C) is incorrect because if there are several beneficiaries, Crummey powers should be given ratably to each. Statement (D) is incorrect because with the way the powers are stated, a beneficiary cannot request an amount greater than the grantor's annual addition.

Which of the following is correct in an Irrevocable Life Insurance Trust (ILIT)? A) The grantor has the ability to reclaim the trust assets. B) The trust is the beneficiary of a life insurance policy. C) The grantor should be the trustee of the ILIT. D) The beneficiary of the ILIT should be the grantor's estate.

B) The trust is the beneficiary of a life insurance policy. In an ILIT, the trust is the direct beneficiary of a life insurance policy. Statement (A) is incorrect because the trust is irrevocable. Statement (C) is incorrect because the grantor will be deceased and cannot serve as the trustee. In addition, naming the grantor as trustee could cause inclusion of the proceeds in the grantor's estate. Statement (D) is incorrect because the beneficiary of an ILIT should not be the grantor's estate, otherwise all estate tax benefits will be eliminated.

All of the following are reasons why trusts are popular asset management tools EXCEPT: A) Trusts can facilitate professional management of trust assets to include the disbursing of those assets. B) Trusts have more favorable income tax rates than individual taxpayers. C) Irrevocable trust can help provide needed liquidity when settling a grantor's estate. D) Revocable trusts can help ease the burden on an executor by retitling assets in the name of the trust while the grantor is alive.

B) Trusts have more favorable income tax rates than individual taxpayers. (A), (C), and (D) are correct statements with regard to trusts. (B) is incorrect as trusts and estates, which share the same tax schedule, have the steepest taxation schedule and planners try to avoid taxation at the trust level if the taxation can be shifted.

Mike created and funded an irrevocable trust for his daughter, Allison, with $1,000,000. Allison was the income beneficiary, and her two daughters (Mike's lineal grandchildren) were the remainder beneficiaries. Which of the following statements is correct? A) If Mike predeceases Allison, this is a skip trust. B) When Allison dies, there is a taxable termination of the trust for GSTT purposes. C) At the inception, this is a skip trust and is subject to GST tax or use of the GSTT exemption when the trust is funded. D) Mike does not need to allocate part of his lifetime GSTT exemption to this transfer, since no GST tax will be due on the transfer.

B) When Allison dies, there is a taxable termination of the trust for GSTT purposes. The transfer is a taxable termination, because when Allison dies, the trust assets will be distributed to skip beneficiaries. Statements (A) and (C) are incorrect because a skip trust is created when the beneficiaries are all skip beneficiaries. Statement (D) is incorrect because the transfer is taxable, and the exemption will be allocated to cover the tax.

Natalie and Ashley own farm land as Joint Tenants with Rights of Survivorship. Natalie contributed $60,000 and Ashley contributed $40,000. The land is currently valued at $1,000,000. If Natalie died today, what amount of the value of the farm land would be included in her gross estate? A) $60,000. B) $500,000. C) $600,000. D) $1,000,000.

C) $600,000. Even though the ownership shares are equal in JTWROS, the actual contribution rule is followed for inclusion in the gross estate for unmarried tenants. Therefore, since Natalie contributed 60% of the property, her estate will include 60% of the Fair Market Value (60% x $1,000,000 = $600,000).

Rachel died in 2021 and her executor is finalizing her estate tax return. The executor has determined that Rachel's adjusted gross estate is $15,830,000 and that her estate is entitled to a charitable deduction in the amount of $500,000. The applicable credit for 2021 is 4,625,800. Calculate the estate tax liability for Rachel's estate. Remember that the estate tax liability = $345,800 + (40% x the taxable estate - $1million). A) $0 B) $1,226,200 C) $1,452,000 D) $6,077,800

C) $1,452,000 Subtract the charitable deduction from the adjusted gross estate to get the taxable estate ($15,830,000 - $500,000 = $15,330,000). The tentative tax on the taxable estate is $6,077,800 ($345,800 on the first million + (0.40 x $14,330,000)). Subtract the applicable estate tax credit to determine the federal estate tax liability ($6,077,800 - $4,625,800 = $1,452,000).

Stephen died in 2021 and his executor is finalizing his estate tax return. The executor has determined that Stephen's adjusted gross estate is $17,430,000 and that his estate is entitled to a charitable deduction in the amount of $1,200,000. The applicable credit for 2021 is 4,625,800. Calculate the estate tax liability for Stephen's estate. Remember that the estate tax liability = $345,800 + (40% x the taxable estate - $1million). A) $0 B) $1,466,200 C) $1,812,000 D) $6,437,800

C) $1,812,000 Subtract the charitable deduction from the adjusted gross estate to get the taxable estate ($17,430,000 - $1,200,000 = $16,230,000). The tentative tax on the taxable estate is $6,437,800 ($345,800 + 0.40 ($15,230,000)). Subtract the applicable estate tax credit to determine the federal estate tax liability ($6,437,800 - $4,625,800 = $1,812,000).

Rita transfers $250,000 of stock to a charitable organization in return for a life annuity on his life valued at $118,000. With regards to this transfer, how much is Rita's charitable deduction? A) $0 B) $118,000 C) $132,000 D) $250,000

C) $132,000 When an individual transfers property in exchange for a charitable annuity, the value of the property less the value of the retained annuity interest is the value of the charitable deduction. In this case, $250,000 - $118,000 = $132,000.

Anthony's grandfather sold the family business, valued at $1,600,000, to him using a private annuity. The private annuity was structured such that Anthony would pay his grandfather $80,000 per year plus interest, for the remainder of his grandfather's life. At the date of the sale, Anthony's grandfather's life expectancy was 20 years and his grandfather was in great health. After eight years, Anthony's grandfather died of a heart attack and Anthony sold the business for $3,000,000 six months after his grandfather's death. What is Anthony's capital gain/loss on the transaction? A) $ 0 B) $1,600,000 C) $2,360,000 D) $3,000,000

C) $2,360,000 A buyer's adjusted basis of property purchased with a private annuity is equal to the sum of all annuity payments paid. In this scenario, Anthony made eight annuity payments of $80,000, or a total of $640,000. Since he sold the property for $3,000,000, his gain is calculated by subtracting his basis from the sales price to arrive at $2,360,000 ($3,000,000 - $640,000).

Cathy and Mark paid $400 for two tickets to the United Church's annual gala ball. The church determined that the fair market value of each ticket was $100. How much can Cathy and Mark deduct on their income tax return? A) $0 B) $100 C) $200 D) $400

C) $200 The value of any tangible benefit received in return for a contribution is not deductible. In this case, Cathy and Mark paid $400 for two tickets and received $200 ($100/ticket) in benefit. The difference, $200 ($400 - $200), is deductible on their income tax return.

Despite his efforts to transfer all of his property out of his estate during his life, Gordon died on January 16th. His family provided the following information about Gordon's assets: Personal Residence: $20,000 (Adjusted Basis); $320,000 (FMV) Rental Property: $84,000 (AB); $80,000 (FMV) Rental Income on above property (February payment): $2,000 (AB) $2,000 (FMV) Canceled vacation cruise refund: (check received 12/31 but not cashed)$4,500 (AB), $4,500 (FMV) Cash: $18,000 (AB); $18,000 (FMV) What is the value of Gordon's gross estate? A) $124,500 B) $128,500 C) $422,500 D) $424,500

C) $422,500 The personal residence, the rental property, the cruise refund and the cash are all included at the fair market value at Gordon's date of death. The rental income is not included because the payment was made after Gordon's death, and it was not owed to Gordon before his death. $320,000 + $80,000 + $4,500 + $18,000 = $422,500. The rental income would be included in the estate's income tax return (Form 1041).

Avi and Chaim are siblings that own a rental property as Joint Tenants with Rights of Survivorship. Avi contributed $75,000 and Chaim contributed $25,000. The land is currently valued at $200,000 and each of them has a 50% interest in the property. If Chaim died today, what amount of the value of the property would be included in his gross estate? A) $25,000. B) $100,000. C) $50,000. D) $200,000.

C) $50,000. Property owned JTWROS follows the actual contribution rule for inclusion in the gross estate unless the tenants are married. Therefore, since Chaim contributed 25% of the property, his estate will include 25% of the Fair Market Value (25% x $200,000 = $50,000).

Steve made the following transfers during the year: • $10,000 to Louisiana State University. The $10,000 contribution allows him to purchase football season tickets. Steve also bought the football season tickets at a cost of $5,000.• $400 to the local public broadcast television station during the annual fund drive. In return for the $400 contribution, Steve received a mug and pen with the station's logo, valued at $8.• 1,000 shares of ABC stock to the United Way. At the date of the contribution, the stock had a fair market value of $50 per share. Steve's adjusted taxable basis in the stock was $10 per share and he held the stock long term. Ignoring any AGI limitations, what is Steve's maximum charitable income tax deduction for this year? A) $18,400 B) $20,392 C) $50,400 D) $58,400

C) $50,400 gnoring any AGI limitations, Steve could not deduct the $10,000 contribution because the contribution entitled him to purchase football tickets. The $400 contribution to the public broadcast television station will not be reduced by the value of the mug and the pen, because those items are considered de minimis. Without any AGI limitations, the full fair market value of the stock contribution may be deductible, $50,000 ($50 x 1,000). Total deduction: $400 + $50,000 = $50,400.

Michael transfers $100,000 of stock to a charitable organization in return for a life annuity on his life valued at $43,000. With regards to this transfer, how much is Michael's charitable deduction? A) $0 B) $43,000 C) $57,000 D) $100,000

C) $57,000 When an individual transfers property in exchange for a charitable annuity, the value of the property less the value of the retained annuity interest is the value of the charitable deduction. In this case, $100,000 - $43,000 = $57,000.

Brody and Tanya recently sold some land they owned for $150,000. They received the land and a check equal to the amount of the then-current annual exclusion five years ago as a wedding gift from Brody's Aunt Jeanette. Aunt Jeanette purchased the land when the property was worth $20,000. One the wedding day, the property was worth $100,000 and Aunt Jeanette paid $47,000 in gift tax. What is the long-term capital gain on the sale of the property? A) $42,400. B) $50,000. C) $92,400. D) $130,000.

C) $92,400. When a donor makes a gift of property other than cash to a donee, the donee will take the property at the donor's adjusted basis including any gift tax paid on the appreciation of the asset. Thus, donee's basis would be: Donor's basis + (Gift tax x % appreciation). In this case, the % appreciation is 80% (80,000/100,000). 80% of the gift tax paid is attributed to the appreciation on the land. So, the equation would be: $20,000 + (47,000 x 80%) = $57,600The gain on the asset would be $150,000 - $57,600 = $92,400. The holding period of the donee will include the holding period of the donor for purposes of subsequent transfers and the determination of long- or short-term capital gains.

Which of the following statements regarding SCINs is correct? A) If the seller outlives the SCIN term, the buyer continues to pay the SCIN payment until the seller's death. B) The payments received by the seller under a SCIN are treated as interest income. C) A SCIN can give the seller a collateral interest in the property sold. D) If the seller dies before the end of the SCIN term, the seller is deemed to have made a taxable gift to the buyer equal to the difference between the payments made and the total principal payments due on the SCIN.

C) A SCIN can give the seller a collateral interest in the property sold. Statement (A) is incorrect because the buyer of a SCIN only makes payments until the earlier of (1) the seller's death, or (2) the term set forth in the SCIN. Statement (B) is incorrect because each payment received by the seller consists of (1) interest income, (2) capital gain, and (3) return of adjusted basis. Statement (D) is an incorrect statement. If the seller dies before the end of the term, the difference between the seller / decedent's adjusted basis and the face value of the note is deemed a transfer of the estate to the buyer and must be included on the estate's fiduciary (income) tax return.

Ralphie, a real estate mogul, dies owning a great deal of real property. Which of the following would be included in Ralphie's probate estate? A) A building owned in sole ownership by Ralphie's wife. Ralphie and his wife do not live in a community property state. B) A vacant lot owned joint tenancy with rights of survivorship by Ralphie and his brother. C) A beach house owned tenancy in common by Ralphie and his mother. D) An office building owned tenancy by the entirety by Ralphie and his wife.

C) A beach house owned tenancy in common by Ralphie and his mother. Statement (A) is incorrect because the property of Ralphie's wife would not be included in his probate estate. Statement (B) is incorrect because property owned JTWROS passes outside of probate. Statement (D) is incorrect because property owned tenancy by the entirety passes outside of probate.

Which of the following statements is not true regarding the need for an estate plan? A) A wealthy, successful business owner who wants to pass his business to hischildren would have complex estate planning needs. B) The sole breadwinner in a family would need an estate plan with significant life and disability insurance coverage. C) A single individual with no family who does not own any assets and lives paycheck to paycheck would not need an estate plan. D) An 80-year old with 5 adult children and 4 grandchildren who just married his third wife would require estate planning for blended families.

C) A single individual with no family who does not own any assets and lives paycheck to paycheck would not need an estate plan. Even individuals without significant assets need an estate plan. The individual in Statement (C) would still need healthcare planning and an executor to oversee his or her final burial wishes. Statement (A) would need an estate plan to determine how to transfer his business in the most cost-efficient manner and minimize estate taxes. Statement (B) would need an estate plan to ensure the individual has sufficient life, long-term care, and disability insurance. Statement (D) would need an estate plan because issues with protection of inheritance often arise in blended families.

All of the following are characteristics of family limited partnerships (FLPs) and why they make excellent transfer tax vehicles EXCEPT: A) Senior family members can retain control of the FLP while they transfer the underlying value of the asset to the younger family members. B) Family values and business/financial lessons gained can be transferred to the recipient family members in the running of the FLP. C) Although they are effective transfer vehicles of family wealth, the structure of FLPs do not offer the general partners (GP) any tools by which they can dissuade the creditors of limited partners (LP) from attaching to the income stream generated by the LP shares. D) FLPs are required to have a valid business purpose to ensure compliance with IRS guidelines.

C) Although they are effective transfer vehicles of family wealth, the structure of FLPs do not offer the general partners (GP) any tools by which they can dissuade the creditors of limited partners (LP) from attaching to the income stream generated by the LP shares. (A), (B), and (D) are correct statements with regard to IDGTs. (D) is incorrect as FLP shares are not effective devices to attach to for creditors of limited partners as the general partners control the income assigned to each share/interest and limited partners have no right to demand payment.

Which of the following statements regarding selling an estate's assets to generate cash is not correct? A) The estate may have income tax consequences. B) The assets may not be sold at full, realizable fair market value. C) Any losses on the sale of the assets are deductible as losses on the estate tax return. D) Any selling expenses are deductible on the estate tax return.

C) Any losses on the sale of the assets are deductible as losses on the estate tax return. Any losses on the sale of the assets are income tax losses and are deductible on the estate's income tax return, not on the estate tax return. All of the other answers are true statements.

Tom loans $5,000 to his daughter Tina. Which of the following is true regarding the loan? A) Interest would not be imputed because the loan is less than the amount of the annual exclusion. B) Interest would not be imputed because loans of $100,000 or less are exempt from both income tax and gift tax consequences. C) Interest would not be imputed because the loan is less than $10,000. D) If Tom forgives the loan, the outstanding loan amount is taxable income to Tina.

C) Interest would not be imputed because the loan is less than $10,000. Statement (A) is incorrect because the annual exclusion is not relevant to imputed interest. Statement (B) is incorrect; loans of less than $10,000 are exempt from both income tax and gift tax consequences. Statement (D) is incorrect because a loan forgiven by family members is considered a gift, not taxable income.

Of the following statements regarding a Qualified Personal Residence Trust (QPRT), which is true? A) At the end of the QPRT term, the residence reverts to the grantor. B) The taxable gift to the remainder beneficiary is eligible for the annual exclusion. C) At the end of the QPRT term, the grantor must begin paying rent to the remainder beneficiaries of the QPRT if he continues to live in the residence. D) A QPRT is ideal for a personal residence that is expected to appreciate at a lower rate than the Section 7520 rate.

C) At the end of the QPRT term, the grantor must begin paying rent to the remainder beneficiaries of the QPRT if he continues to live in the residence. At the end of the QPRT, the grantor must pay rent, or run the risk that the IRS will deem the gift was not actually transferred and the fair market value of the home will be included in the grantor's estate. Statement (A) is incorrect because the residence transfers to the remaindermen at the end of the QPRT term. Statement (B) is incorrect because the remainder interest is a future interest gift and is not eligible for the annual exclusion. Statement (D) is incorrect because the QPRT is ideal for a personal residence that is expected to appreciate at a higher rate than the Section 7520 rate, because the gift tax is assessed at the 7520 rate.

Donna has AGI of $100,000. Donna owns a rare antique with an adjusted basis of $200,000. The antique is currently worth $2,000,000. Assuming that Donna's AGI will remain at $100,000 for the next six years, which of the following would you recommend to her if she donates the antique to a museum this year? A) Donna should deduct the entire fair market value of the antique this year. B) Donna should deduct $30,000 this year and every year for the next five years. C) Donna should deduct $50,000 this year and for the next four years. D) Donna should deduct $200,000 this year.

C) Donna should deduct $50,000 this year and for the next four years. Statement (C) is correct because, given Donna's AGI, she will obtain the maximum tax benefit by electing to deduct the adjusted basis of the antique, subject to a ceiling of 50% of her AGI. Electing to deduct the adjusted basis produces a deduction of $50,000 per year for four years, for a total charitable deduction of $200,000 (equals basis). Statement (B) is incorrect because if Donna elects to deduct the fair market value of the antique, she is limited to 30% of her AGI, her deduction will be $30,000 per year for six years, or a total of $180,000, which is less advantageous than statement (C). Donna may not take the deductions described in statement (A) and statement (D).

Which of the following describes second-to-die life insurance? I. It is generally not includible in any insured's gross estate if it is owned in an ILIT. II. It can provide liquidity to pay estate taxes at the death of the second insured. III. It pays a partial benefit at the death of the first to die (administrative and estate taxes), with the remainder paid in full at the second death. IV. Premiums are usually less expensive than for individual policies on each of the two insureds for the same face amount. A) I and II B) III and IV C) I, II and IV D) I, II, III and IV

C) I, II and IV Statement III is incorrect because survivorship life pays the entire death benefit at the second death. All of the other statements are true.

Which of the following is/are considered a disadvantage(s) of probate? I. The process can result in delays. II. The process may be expensive. III. The process does not provide clear title to heirs and legatees. IV. The process is open to public scrutiny. A) I only B) I and II C) I, II, and IV D) I, II, III, and IV

C) I, II, and IV An advantage of probate is that it provides clear title to heirs and legatees. All of the other options are disadvantages of the probate process.

Which of the following techniques can be used to lower the value of an individual's gross estate? I. A Totten Trust II. A Qualified Personal Residence Trust III. A Family Limited Partnership IV. An Irrevocable Life Insurance Trust A) I only B) I and II C) II, III, and IV D) I, III, and IV

C) II, III, and IV Totten trusts are used to avoid probate, not to lower the value of the gross estate. All of the other techniques can be used to lower the value of an individual's gross estate.

Which of the following is/are considered real property? I. Bonds. II. Patents. III. Hot water tank inside of a residential home. IV. Land held for investment. A) III only B) I and II C) III and IV D) I, II, and III

C) III and IV Real property is land and buildings. A hot water tank affixed inside of a home and land for investments are tangible real property. The nature of the property in the hands of the owner (investment, personal, inventory) does not affect the type of property. Bonds and patents are intangible property.

Laurie and Chance are considering purchasing a piece of land on which they plan to build a vacation home. Laurie and Chance are engaged to be married, and are unsure of how they should title the property. Which of the following statements is correct regarding their ownership and titling of the land? A) Laurie and Chance cannot own the property as joint tenants because joint tenancies may only be established between married parties. B) If Laurie and Chance were married and owned the property in tenancy by the entirety, one-half of the value of the property will be included in the probate estate of the first spouse to die without regard to the actual contribution of each spouse. C) If the property is held as a joint tenancy with right of survivorship then Laurie and Chance will each own the same fractional share in the property regardless of how much they contribute. D) If the property is held as a joint tenancy with right of survivorship and Chance dies first, the property will pass to Laurie unless Chance's will directs a different disposition.

C) If the property is held as a joint tenancy with right of survivorship then Laurie and Chance will each own the same fractional share in the property regardless of how much they contribute. Joint tenancy with right of survivorship requires equal ownership. Statement (A) is incorrect because joint tenancies may be established by spouses or non-spouses. Statement (B) is incorrect because in tenancies by the entirety, each would be deemed to have contributed 50%, therefore only 50% would be included in the gross estate of the first spouse to die. Nothing will be included in the probate estate. Statement (D) is incorrect because if the property is held as JTWROS, then the property will transfer automatically at the first tenant's death, regardless of what the will dictates.

Which of the following empowers an executor to act as the agent of a probate court? A) Surety Bond B) Letters of Administration C) Letters Testamentary D) Intestacy Laws

C) Letters Testamentary Statement (A) is the bond that an administrator must generally post. Statement (B) is what empowers an administrator to act as the agent of a probate court. Statement (D) describes the state laws that govern the disposition of a decedent's estate if he has failed to prepare a valid will.

Which of the following is an advantage of dying intestate? A) Liquidity can be easily generated by selling small business ownership interests. B) Estate property can be designated to charity and friends. C) Minor children and surviving spouses will get priority with distributing estate assets. D) Only the very wealthy need estate planning services, so money can be saved by avoiding the creation of a will.

C) Minor children and surviving spouses will get priority with distributing estate assets. State intestacy laws distribute most or all of an estate to the surviving spouse and minor children. Statement (A) is incorrect because small businesses are not easily liquidated and may be liquidated for much less than fair market value. Statement (B) is incorrect because intestacy laws only allow for distribution to family members. Statement (D) is incorrect because most people need estate planning services to ensure their wishes are protected and transfer costs are minimized.

Nellie recently executed a power of attorney giving Jessie the power to perform certain tasks. Which of the following powers given to Jessie would cause the power to be deemed a general power of appointment? A) Nellie gave Jessie the power to use Nellie's money to pay Nellie's creditors. B) Nellie gave Jessie the power to sell and buy property on Nellie's behalf. C) Nellie gave Jessie the power to use Nellie's money to pay Jessie's creditors. D) Nellie gave Jessie the power to make gifts to Nellie's heirs and charities.

C) Nellie gave Jessie the power to use Nellie's money to pay Jessie's creditors. Giving Jessie the power to pay his own creditors creates a general power of appointment over the assets. The other powers do not benefit Jessie and thus do not create a general power of appointment.

You are a CFP® professional and although you never went to law school, you consider yourself to be very good at reviewing wills. Your client, Catherine, asks you to prepare a will for her. Should you prepare a will for Catherine? A) Yes, Catherine is your best client and you might lose her if you do not prepare the will. B) Yes, it is permissible for a CFP® professional to prepare a legal document. C) No, preparing Catherine's will would be considered the unauthorized practice of law. D) No, you should only prepare Catherine's will if you are going to be a beneficiary under the will.

C) No, preparing Catherine's will would be considered the unauthorized practice of law. Drafting legal documents, such as wills, is an activity reserved for licensed attorneys. If you are not a licensed attorney and you prepare a legal document, you have engaged in the unauthorized practice of law.

Which of the following is not a correct statement with respect to the unlimited marital deduction? A) The recipient spouse must be a US citizen at the time the transfer occurs through gift or at death. B) To get the estate tax marital deduction for assets passing at death, the property must have been included in the gross estate of the deceased spouse. C) Property passing to an ex-spouse upon the death of a former spouse will qualify for the deduction assuming all other requirements are met and the decedent and the recipient were married at some point during the year of death. D) The deduction may be taken only for assets which the surviving spouse actually receives.

C) Property passing to an ex-spouse upon the death of a former spouse will qualify for the deduction assuming all other requirements are met and the decedent and the recipient were married at some point during the year of death. To qualify for the marital deduction, spouses must be married at the time of death or gift, even if legally separated. The other answers are all correct statements.

Which of the following is true regarding the gift tax return? A) The due date for the gift tax return is 9 months after the gift is made. B) During the year, James gifts his son $10,000. James must file a gift tax return. C) Renata gifts her granddaughter a $36,000 tuition payment, made directly to her university. Renata does not need to file a gift tax return. D) Frankie gifts his wife a surprise trip to the Bahamas, worth $17,000. He must file a gift tax return.

C) Renata gifts her granddaughter a $36,000 tuition payment, made directly to her university. Renata does not need to file a gift tax return. Gift tax returns do not need to be filed for qualified transfers. Statement (A) is incorrect because the gift tax return must be filed by April 15 of the year following the gift. Statement (B) is incorrect because the return does not need to be filed for gifts under the annual exclusion. Statement (D) is incorrect because gifts between spouses do not need to be reported.

Which of the following is a feature of a testamentary trust? A) Created and funded by the grantor while they are still alive. B) Shifts the income tax burden to a lower-bracket taxpayer. C) Results in the inclusion of assets in the gross estate. D) Avoids probate.

C) Results in the inclusion of assets in the gross estate. Assets in a testamentary trust come from the estate assets. Therefore, they are included in the gross estate. Statement (A) is incorrect because testamentary trusts are not funded until after the grantor's death. Statement (B) is incorrect because testamentary trusts do not facilitate income tax shifting. Statement (D) is incorrect because testamentary trusts are created under the will, and are therefore part of probate assets.

Although he has a vast fortune, Ricky has decided not to prepare an estate plan because he believes that his surviving family members will divide up his assets appropriately. Which of the following is not a risk associated with failing to plan an estate? A) Ricky's estate could incur excessive transfer taxes. B) Ricky's favorite Corvette may not be transferred to his ex-wife, Carla. C) Ricky's insurance policy on his own life may not be paid out to the named beneficiary. D) Ricky's current wife, Lucille, may not provide for Ricky's children from a previous marriage.

C) Ricky's insurance policy on his own life may not be paid out to the named beneficiary. The proceeds of insurance policies with named beneficiaries pass outside of probate via state contract law. Ricky's failure to plan his estate will not affect the payment of his insurance policy.

All of the following statements regarding powers of attorney are correct EXCEPT? A) A general power of attorney may or may not be durable. B) A springing power of attorney is a useful tool to deal with the contingency of the principal's potential incapacity. C) Special powers of attorney give the agent unlimited power to deal with the principal's affairs. D) Estate tax inclusion for the agent is a potential disadvantage to the agent of holding such a power.

C) Special powers of attorney give the agent unlimited power to deal with the principal's affairs.

Of the following purported benefits of using a bypass trust in conjunction with the use of the unlimited marital deduction, which is not a correct statement? A) A bypass trust allows any appreciation in an asset that occurs after the transfer to the bypass trust to escape estate taxation at the death of the second spouse to die. B) A surviving spouse can be a beneficiary of the bypass trust although often any distributions would be based on the HEMS standard. C) The bypass trust qualifies for the marital deduction as asset from the bypass trust are included in the estate of the surviving spouse at their death. D) Creditor protection for beneficiaries is potentially available in a bypass trust whereas creditors of the surviving spouse may have access to assets passing directly to a surviving spouse.

C) The bypass trust qualifies for the marital deduction as asset from the bypass trust are included in the estate of the surviving spouse at their death. Option C is incorrect because a bypass trust is designed to utilize the utilize the estate tax credit of the first spouse to die. Assets in the bypass trust are designed to not be included in the estate of the surviving spouse so that spouse's beneficial interest in the trust must be properly limited. Contrast that to GPOA or QTIP trusts. The other statements are all correct statements with respect to bypass trusts.

Which of the following statements is true regarding the marital deduction? A) The availability of the unlimited marital deduction eliminates the estate tax due for both the decedent spouse to die and the surviving spouse. B) Marital deduction property can consist of estate property as well as property that is not within the gross estate of the decedent spouse. C) The death benefit of a life insurance policy included in a decedent's gross estate is eligible for the unlimited marital deduction if the surviving spouse is the listed beneficiary and receives the proceeds. D) The decedent spouse must be a US Citizen in order to utilize the marital deduction.

C) The death benefit of a life insurance policy included in a decedent's gross estate is eligible for the unlimited marital deduction if the surviving spouse is the listed beneficiary and receives the proceeds. If the death benefit of a life insurance policy is included in a decedent's gross estate, and the surviving spouse is the listed beneficiary and receives the proceeds, the value of the death benefit will be eligible for the unlimited marital deduction. Statement (A) is incorrect because the marital deduction only defers the payment of estate tax for the surviving spouse. Statement (B) is incorrect because marital deduction property must be included in the gross estate of the decedent spouse. Statement (D) is incorrect because the decedent spouse does not need to be a US Citizen, only the surviving spouse must meet this requirement.

Which of the following statements concerning trust formation is correct? A) The trustee of the trust will receive the trust corpus after paying the income to the income beneficiary. B) The remainder beneficiary of a trust receives an annuity payment each year. C) The grantor of a trust contributes property to a trust, which will be managed by the trustee. D) The income beneficiary of a trust always receives the trust property at the termination of the trust.

C) The grantor of a trust contributes property to a trust, which will be managed by the trustee. Grantors contribute assets to a trust, which are managed by the trustee. Statements (A), (B), and (D) are incorrect as the remainder beneficiary of a trust receives the trust corpus when the trust terminates, and the income beneficiary receives trust distributions during the trust term.

An aunt and her nephew own a rental property as tenants in common valued at $200,000. The aunt owns an 80% interest and her nephew owns a 20% interest. The aunt is elderly and in failing health. She wants to sell the property to get cash to pay off her medical bills. What impact will her sale have on her nephew? A) The nephew will have to agree to the sale. B) The aunt can sell 100% of the property and reimburse the nephew for his loss. C) The nephew will become co-tenants with the person the aunt sells to. D) The nephew will lose his right of survivorship.

C) The nephew will become co-tenants with the person the aunt sells to. Statement (C) is correct because the aunt can sell her interest to another individual, who becomes a co-tenant. Statement (A) is incorrect because a tenant in common does not need permission from the other tenants to dispose of their property interest. Statement (B) is incorrect because the aunt cannot sell a greater share of the property than she owns. Statement (D) is incorrect because there is no right of survivorship with tenancy in common.

Which of the following is not a requirement for a general power of appointment trust to be eligible for the unlimited marital deduction? A) No person, other than the surviving spouse, may appoint any part of the trust property to anyone other than the surviving spouse. B) The general power of appointment granted to the surviving spouse must be exercisable by the surviving spouse alone. C) The surviving spouse's right to the trust property must be limited to an ascertainable standard, such as health, education, maintenance, and support. D) The surviving spouse must be entitled to receive all of the income from the trust, at least annually.

C) The surviving spouse's right to the trust property must be limited to an ascertainable standard, such as health, education, maintenance, and support. Statements (A), (B), and (D) are the requirements for a general power of appointment trust to be eligible for the unlimited marital deduction. Statement (C) does not describe a requirement of a general power of appointment trust eligible for the marital deduction.

Which of the following accurately describes a QTIP Trust? A) A QTIP is sometimes called a "B" or "Q" Trust. B) Trust income must be paid to the spouse or other designated beneficiary at least annually. C) The trust assets will be included in the gross estate of the surviving spouse. D) The surviving spouse designates the remainder beneficiaries of the QTIP.

C) The trust assets will be included in the gross estate of the surviving spouse. Statement (A) is incorrect because a QTIP is not the same as a "B" trust. Statement (B) is incorrect because the trust's income must be paid only to the spouse, not to any other beneficiary. Statement (D) is incorrect because the surviving spouse does not choose the remainder beneficiaries of the QTIP. Remainder beneficiaries are determined by the grantor spouse.

Which of the following are methods is the best method for transferring property outside of probate? A) Verbally agreeing to keep separate finances, even though the couple lives in a community property state. B) Transfer of estate property through a testamentary trust C) Titling property as joint tenants with right of survivorship D) Relying on the state's intestacy statutes for distribution of the estate property

C) Titling property as joint tenants with right of survivorship JTWROS property is non-probate property, because the property is transferred to the surviving tenants by operation of law. (A) is incorrect because although the couple may verbally agree, all marital property earned or acquired is assumed to be marital property. A written agreement would be a better option to secure the property as separate. (B) is incorrect because testamentary trust property are probate property. (D) is incorrect because intestate property is considered probate property since it is administered through the probate court system.

At the request of her financial advisor, Heba is making a list of her estate planning goals. She is a 68-year old widow with a net worth is $24 million. She has 3 adult children and 5 grandchildren. Which of the following goals would be the least important in developing her estate plan? A) To minimize estate and transfer taxes B) To avoid unnecessary fees and costs C) To provide financial support for her children D) To plan for her incapacity

C) To provide financial support for her children Providing financial support for children is a critical estate planning goal for minor children, but takes less importance for adult children. Options (A), (B), and (D) are all important reasons she should have an estate plan.

An estate tax return must be filed for a U.S. resident or a U.S. citizen who dies during 2021 if the total value of his gross estate plus post-1976 adjusted taxable gifts on his date of death is greater than: A) $15,000 B) $1,000,000 C) $4,625,800 D) $11,700,000

D) $11,700,000 This question is asking for the federal exemption amount for 2021, or the fair market value of property that can transfer with an estate tax less than or equal to the applicable estate tax credit. For 2021, the exemption amount is $11,700,000. Statement (A) refers to the annual exclusion. Statement (B) is the tax on the first million dollars of taxable gifts and estate. Statement (C) is the credit equivalency for the federal exemption amount.

In 2020, Lori assigned a paid-up whole life insurance policy to an Irrevocable Life Insurance Trust (ILIT) for the benefit of her three children. The ILIT contained a Crummey provision for the benefit of each child. At the time of the transfer, the whole life insurance policy was valued at $200,000, and since Lori had not made any other taxable gifts during her lifetime, she did not owe any gift tax. Lori died in 2021, and the face value of the whole life insurance policy of $2,000,000 was paid to the ILIT. Regarding this transfer, how much is included in Lori's gross estate at her death? A) $0 B) $164,000 C) $964,000 D) $2,000,000

D) $2,000,000 The death benefit of a life insurance policy transferred within three years of the decedent's date of death is included in the decedent's gross estate. In this case, Lori transferred the policy one year before her death, so the full death benefit of $2,000,000 is included in her gross estate.

This year, Dottie donated $10,000 in cash to her church, and she also donated medical supplies with a fair market value and adjusted basis of $15,000 to the Red Cross. Dottie's AGI for this year is $50,000. What is Dottie's charitable income tax contribution deduction for 2021? A) $10,000 B) $15,000 C) $20,000 D) $25,000

D) $25,000 The deduction of charitable donations in the form of cash is limited to 60% of AGI, but a provision in the CARES Act has increased this to 100% for 2021. Dottie's AGI is $50,000, so the deduction of any cash donations to a public charity will be limited to $30,000. The deduction of charitable donations of ordinary income property is limited to the lesser of the adjusted basis or the fair market value of the property, up to 50% of AGI. Dottie has made a total donation of $25,000 this year, and her deduction will be limited to $25,000.

In 2020, Grace assigned a paid-up whole life insurance policy to an Irrevocable Life Insurance Trust (ILIT) for the benefit of her four children. The ILIT contained a Crummey provision for the benefit of each child. At the time of the transfer, the whole life insurance policy was valued at $350,000, and since Grace had not made any other taxable gifts during her lifetime, she did not owe any gift tax. Grace died in 2021, and the face value of the whole life insurance policy of $4,000,000 was paid to the ILIT. Regarding this transfer, how much is included in Grace's gross estate at her death? A) $0 B) $290,000 C) $350,000 D) $4,000,000

D) $4,000,000 The death benefit of a life insurance policy transferred within three years of the decedent's date of death is included in the decedent's gross estate. In this case, Grace transferred the policy one year before her death, so the full death benefit of $4,000,000 is included in her gross estate.

This year, Freddy donated $20,000 in cash to her church, and she also donated medical supplies with a fair market value and adjusted basis of $25,000 to the Salvation Army. Freddy's AGI for this year is $100,000. What is Freddy's charitable income tax contribution deduction for 2021? A) $20,000 B) $25,000 C) $60,000 D) $45,000

D) $45,000 The deduction of charitable donations in the form of cash is limited to 60% of AGI, but a provision in the CARES Act has increased this to 100% for 2021. Freddy's AGI is $100,000, so the deduction of any cash donations to a public charity will be limited to $100,000 for 2021. The deduction of charitable donations of ordinary income property is limited to the lesser of the adjusted basis or the fair market value of the property, up to 50% of AGI. Freddy has made a total donation of $45,000 this year, and her deduction will be limited to $45,000.

The Generation Skipping Transfer Tax (GSTT) has all the following characteristics, except: A) GSTT gifts to direct skips qualifying for the annual exclusion are not subject to the tax. B) Assets transferred to a trust that has a grandchild as the sole beneficiary may be subject to both gift and generation skipping transfer tax. C) If all the beneficiaries of a trust are grandchildren (all whose parents are living) of the grantor then the trust is subject to GSTT. D) A "skip person" is a person who is one or more generations younger than the transferor.

D) A "skip person" is a person who is one or more generations younger than the transferor. Statements (A), (B), and (C) are true but in the case of statement (D), a grandchild whose parent has died has moved up a generation regarding skip-person considerations. A skip beneficiary is generally a person who is two or more generations younger than the grantor.

Which of the following charitable remainder trusts would be appropriate? A) A CRAT for an individual who wishes to make annual contributions to his alma mater. B) A CRUT for a widow who is on a fixed budget. C) A CRAT for a business owner who wants to transfer the business to his children. D) A CRAT for a wealthy individual who wants to remove appreciating property from her estate.

D) A CRAT for a wealthy individual who wants to remove appreciating property from her estate. Statement (D) will allow the individual to freeze the asset value in her estate at the time of the charitable transfer. Statement (A) is incorrect because a CRAT does not allow additional contributions. Statement (B) is incorrect because a CRUT has variable payments that would not provide predictability for an individual on a fixed budget. Statement (C) is incorrect because a CRAT is a means of transferring to charity, not to family members.

Colin would like to use his recent inheritance of $200,000 to establish a charitable remainder trust. Colin would like to have the flexibility to make additional contributions to the charitable remainder trust in the future. Which of the following would you recommend for Colin? A) A Charitable Remainder Annuity Trust B) A Charitable Gift Annuity C) A Charitable Lead Unitrust D) A Charitable Remainder Unitrust

D) A Charitable Remainder Unitrust Option a is incorrect because additional contributions may not be made to a CRAT. Option c is incorrect because a CLUT is not a charitable remainder trust. Option b is incorrect because each donation is a separate annuity and the annuity it not a remainder trust.

Janet would like to use her recent inheritance of $350,000 to establish a charitable remainder trust. She would like to have the flexibility to make additional contributions to the charitable remainder trust in the future. Which of the following would you recommend for Janet? A) A Charitable Remainder Annuity Trust B) A Charitable Gift Annuity C) A Charitable Lead Unitrust D) A Charitable Remainder Unitrust

D) A Charitable Remainder Unitrust Option a is incorrect because additional contributions may not be made to a CRAT. Option c is incorrect because a CLUT is not a charitable remainder trust. Option b is incorrect because each donation is a separate annuity and the annuity it not a remainder trust.

Of the following expenditures from an estate, which is not a deduction from the gross estate or adjusted gross estate to arrive at the taxable estate? A) Payment to United Charitable Organization (a charity qualifying under IRC Section 501(c)(3)) to satisfy a specific bequest. B) Distribution of assets to spouse to satisfy specific bequests listed in will. C) Payment to Second USA Bank for an outstanding credit card balance. D) A payment to decedent's friend for $10,000 to satisfy a specific bequest.

D) A payment to decedent's friend for $10,000 to satisfy a specific bequest. Payments made to satisfy specific bequests to individuals other than a surviving spouse or a charity are not deductions from the gross estate to arrive at the taxable estate. All of the others are deductible expenses or transfers.

Your client, Albert, is 68 years old. He is interested in establishing a trust with a value of $6,000,000 for his family. He is aware of the Generation Skipping Transfer Tax, and he has asked you for your advice as to which of the following would be considered a skip person. Which of the following is a skip person? A) Albert's son, Patrick, who is age 17. B) Albert's grandson, Connor, age 14, whose mother (Albert's daughter) died in an auto accident this year. C) Albert's mother, Thelma. D) A trust that Albert had established 3 years ago for Albert's favorite employee, Sam, who has just turned 20.

D) A trust that Albert had established 3 years ago for Albert's favorite employee, Sam, who has just turned 20. Due to the age difference of more than 37½ years and the non-related party status, the trust for Sam is a skip person. The reason Patrick is not a skip person is because he is a first-generation descendant. Connor is not a skip person because his mother's death moves him up a generation (predeceased parent rule). Albert's mother is older than him, so she is not a skip person.

Colleen transferred ownership of a whole life insurance policy on her life to an Irrevocable Life Insurance Trust (ILIT) six years ago and retained the right to borrow against the policy. When Colleen dies, the proceeds of the life insurance policy are: A) Included in Colleen's federal gross estate if she has any outstanding loans against the life insurance policy. B) Included in Colleen's federal gross estate if Colleen continued paying the policy premiums after the life insurance policy was transferred to the ILIT. C) Never included in Colleen's federal gross estate. D) Always included in Colleen's federal gross estate.

D) Always included in Colleen's federal gross estate. The IRC (Section 2042) states that if a decedent owns a life insurance policy on her own life or possesses any incidents of ownership in the policy on the date of her death, the policy death benefit will be included in her gross estate. The right to borrow against the life insurance policy is considered an incident of ownership, which would cause inclusion in Colleen's federal gross estate. Because Colleen retained this right, her federal gross estate would include the death benefit of the whole life insurance policy owned by the ILIT. Statement (A) is incorrect because as long as the decedent has the right to take a loan against the policy, it is considered an incident of ownership. There is no requirement that a loan be outstanding. Statement (B) is incorrect because paying the premium on a policy transferred to an ILIT does not create the incidents of ownership that would cause inclusion in Colleen's federal gross estate.

Andy's net worth is $18,500,000, consisting entirely of his separate property. His wife's net worth is $500,000, consisting entirely of her separate property. As part of Andy's estate plan, he would like to transfer as many assets to his wife as possible, while making the most of his applicable estate tax credit. He would also like to ensure that his wife has access to all of his net worth for the rest of her life. It is his wish that at his wife's death, the children will inherit whatever is left, with the least possible amount of transfer taxes owed. Which of the following estate plans would fulfill Andy's goals? Assume that Andy died in 2021, and that he had not used any of his exemption amount. A) Andy's estate could transfer $11,200,000 to Andy's wife as a specific bequest and direct his estate's executor to donate Andy's entire unused exemption to his wife. B) At Andy's death, his estate could create an ILIT for the sole benefit of Raymond's children by utilizing Andy's remaining applicable estate tax credit equivalency. These funds could be used to purchase life insurance on Andy's wife. Any remaining assets should be given the Andy's wife outright. C) At Andy's death, his estate should transfer $11,700,000 to an Irrevocable Trust for the sole benefit of Andy's children. The remaining assets should be given outright to Andy's wife. D) At Andy's death, $11,700,000 should be transferred to a bypass trust for the benefit of Andy's children. Andy's wife should be given the power to invade the bypass trust for an ascertainable standard. The remaining assets should be given outright to Andy's wife.

D) At Andy's death, $11,700,000 should be transferred to a bypass trust for the benefit of Andy's children. Andy's wife should be given the power to invade the bypass trust for an ascertainable standard. The remaining assets should be given outright to Andy's wife. Only statement (D) would give Andy's wife access to all of his estate, and utilize his applicable estate tax credit; therefore, statement (D) is the best option. Statement (A) is not the best option, because we do not know what the state of the law of portability will be at Andy's wife's death. And, while statement (A) assumes that Andy's wife's now ten million dollar estate will pass to her beneficiaries with no estate tax, she could lose Andy's unused exemption amount by remarrying and becoming the surviving spouse of another. Moreover, by giving all of his fortune outright to his wife, it is possible that none will go to his children. Statement (B) may maximize future estate benefits for Andy's children after Andy's wife's death but does not meet Andy's desires to allow his wife to access all of the funds. Andy's wife would not have access to the funds within the ILIT. Statement (C), like statement (B), does not meet Andy's goals because Andy's wife would not have the right to access the funds within the Irrevocable Trust.

Raymond's net worth is $15,000,000, consisting entirely of his separate property. His wife's net worth is $200,000, consisting entirely of her separate property. As part of Raymond's estate plan, he would like to transfer as many assets to his wife as possible, while making the most of his applicable estate tax credit. He would also like to ensure that his wife has access to all of his net worth for the rest of her life. It is his wish that at his wife's death, the children will inherit whatever is left, with the least possible amount of transfer taxes owed. Which of the following estate plans would fulfill Raymond's goals? Assume that Raymond died in 2021, and that he had not used any of his exemption amount. A) Raymond's estate could transfer $10,000,000 to Raymond's wife as a specific bequest and direct his estate's executor to donate Raymond's entire unused exemption to his wife. B) At Raymond's death, his estate could create an ILIT for the sole benefit of Raymond's children by utilizing Raymond's remaining applicable estate tax credit equivalency. These funds could be used to purchase life insurance on Raymond's wife. Any remaining assets should be given the Raymond's wife outright. C) Raymond should transfer $11,700,000 to an Irrevocable Trust for the sole benefit of Raymond's children and his wife today. D) At Raymond's death, $11,700,000 should be transferred to a bypass trust for the benefit of Raymond's children. Raymond's wife should be given the power to invade the bypass trust for an ascertainable standard. The remaining assets should be given outright to Raymond's wife.

D) At Raymond's death, $11,700,000 should be transferred to a bypass trust for the benefit of Raymond's children. Raymond's wife should be given the power to invade the bypass trust for an ascertainable standard. The remaining assets should be given outright to Raymond's wife. Only statement (D) would give Raymond's wife access to all of his estate, and utilize his applicable estate tax credit; therefore, statement (D) is the best option. Statement (A) is not the best option, because the portability rules may change after Raymond's death, and because his wife may remarry. Moreover, by giving all of his fortune outright to his wife, it is possible that none will go to his children if their relationship changes over time. Statement (B) may maximize future estate benefits for Raymond's children after Raymond's wife's death but does not meet Raymond's desires to allow his wife to access all of the funds because she would not have access to the funds within the ILIT. Statement (C), is not a good option because Raymond would lose access to his assets immediately.

Michael has a wife, Diane, and two children. Michael's daughter, Maureen, graduated from college in May and began a career as a teacher. In 2021, Michael rewarded Maureen for her hard work and purchased a new car for her that cost $21,000. The funds to purchase the car came from an account that is part of Michael's separate assets (not marital). Since the gift exceeded the gift tax annual exclusion amount of $15,000, the excess of $6,000 will be a taxable gift and will reduce Michael's applicable gift tax credit amount. How can Diane help Michael avoid making a taxable gift? A) Diane can elect to split the gift with Michael by making the $6,000 portion of the taxable gift a gift attributable to Diane. B) Diane can choose to pay any applicable gift tax upon Michael's death under gift splitting C) Diane can't help Michael since the car was purchased with separate assets. D) Diane can elect to split the gift with Michael for Michael's final tax year, attributing $10,500 of the gift to Diane.

D) Diane can elect to split the gift with Michael for Michael's final tax year, attributing $10,500 of the gift to Diane. Gift splitting means splitting the gift and making half attributable to each spouse for the election year. Statement (A) is incorrect because gift splitting doesn't mean taking over their taxable gift amount. Statement (B) is incorrect because this is not what gift splitting means. Statement (C) is incorrect because Diane can help Michael despite his use of separate assets.

Elizabeth, a widow, has decided to set up trusts for each of her four grandchildren to take advantage of the generation-skipping transfer tax exemption. In 2021, she gives each grandchild $70,000. If Elizabeth has not made any previous taxable gifts, which of the following is true? A) Her taxable gifts total $265,000 and she will owe $106,400 in gift tax. B) Her taxable gifts total $280,000 but she will owe no gift tax. C) Her taxable gifts are $0 and she will owe no gift tax. D) Her taxable gifts are $220,000 and she will owe no gift tax.

D) Her taxable gifts are $220,000 and she will owe no gift tax. Her taxable gifts are the total gifts less the 4 annual exclusions ($280,000 - $60,000 = $220,000). However, since she has made no taxable gifts before, her gift tax owed will be zero since she can utilize her GST exemption to cover the tax.

Death benefit proceeds from a life insurance policy are included in a decedent's gross estate in which of the following circumstances: I. The decedent gave the policy to his father four years ago, but retained the right to change the name of the beneficiary. II. The policy beneficiary is a grantor trust of the decedent, but the policy is owned by a closely held corporation. III. The decedent gave the policy to a charity seven years ago. IV. The decedent transferred the policy to an irrevocable life insurance trust five years ago with no retained incidents of ownership. A) I and IV B) II and III C) III and IV D) I and II

D) I and II Statement I is included because the decedent retained an ownership right. Statement IV is not included because the policy was transferred more than three years ago.

Some reasons to use life insurance to fund business continuation agreements include which of the following: I. It provides sufficient assets for the buyer to perform on the contract. II. Insurance protects the company and its shareholder because the IRS cannot challenge the value of stock if provided for in a Shareholders Agreement (SHA). III. Funding with insurance ensures the agreement will be legally enforceable. IV. The insurance provides tax-advantaged dollars at the time when the agreement is triggered. A) IV only B) II and IV C) I, II and III D) I and IV

D) I and IV Statement II is incorrect because the IRS may challenge the valuation of stock in business continuation agreements, with or without insurance; the IRS is not bound by any contractual agreement between the company and its shareholders. Statement III is incorrect because the agreement is legally enforceable with or without insurance. The other two statements are correct.

Which of the following are characteristics of a qualified disclaimer? I. The disclaimant may not direct the bequest to another person selected by the disclaimant. II. The disclaimer must be irrevocable. III. The disclaimer must be received by the executor of the estate in writing within nine months of the death of the decedent. IV. The disclaimant may disclaim any percent or fraction of an asset. A) I and II B) I, II, and III C) I, III, and IV D) I, II, III, and IV

D) I, II, III, and IV A qualified disclaimer must be written, irrevocable, and received by the executor of the estate within 9 months. It must not direct the asset and can be for any interest, either partial or full.

Which statement(s) is/are true for Generation-Skipping Transfer Tax (GSTT)? I. Applies to transfers to persons who are two or more generations lower than the transferor. II. Gifts to unrelated individuals are exempt from the GSTT. III. There is an $11,700,000 lifetime exemption in 2021 for GSTT. IV. Transfers qualifying for gift tax annual exclusion generally are also excluded from GSTT. A) IV only B) I and IV C) II and III D) I, III, and IV

D) I, III, and IV Statement II is incorrect because gifts to family members as well as gifts to unrelated individuals are subject to the GSTT.

An intentionally defective grantor trust (IDGT) can be set up for all of the following tax-efficient reasons EXCEPT: A) Having the grantor(s) retain the liability of the income taxes due on the assets in trust will help "burn down" the grantor's assets for transfer tax purposes. B) The grantor can substitute property in trust without causing inclusion in the grantor's gross estate. C) By creating an installment sale to an IDGT, a grantor can "freeze" the transfer tax value of the transferred asset at the date of transfer. D) IDGT can be revocable trust and still accomplish their desired income and transfer tax objectives.

D) IDGT can be revocable trust and still accomplish their desired income and transfer tax objectives. (A), (B), and (C) are correct statements with regard to IDGTs. (D) is incorrect as transfer tax objectives will not be accomplished by the use of a revocable trust.

Which of the following is not a benefit of an Intentionally Defective Grantor Trust (IDGT)? A) An IDGT allows more trust income to be taxed at the grantor's lower individual rates compared to trust income tax rates. B) IDGTs enable the grantor to remove assets from her taxable estate. C) A grantor may sell an appreciating asset to the IDGT without owing any capital gains tax or tax on interest income. D) IDGTs allow a grantor to reclaim the assets at any time.

D) IDGTs allow a grantor to reclaim the assets at any time. IDGTs are irrevocable and assets cannot be reclaimed. All of the other statements are true.

Jack purchases a life insurance policy on his own life and names his three children as beneficiaries. Which of the following is true? A) The premium payments Jack makes are taxable gifts to his children. B) If Jack sells the policy to his children, the proceeds are subject to gift tax. C) If Jack transfers the policy to an ILIT, the transfer is not subject to gift tax. D) If Jack transfers ownership of the policy to his children, the transfer is a taxable gift.

D) If Jack transfers ownership of the policy to his children, the transfer is a taxable gift. Transfer of policy ownership is a taxable gift. Statement (A) is incorrect because premium payments are not taxable gifts if made by the owner of the policy. Statement (B) is incorrect because sales are not subject to gift tax. Statement (C) is incorrect because transfer of an existing policy to an ILIT is a taxable gift.

Marie is the founder and sole owner of Purple Cakes Bakery. Allen has offered to buy her business for a price Marie considers reasonable, but Allen does not have all of the funds necessary to pay for the business at the current time. Marie wants to accept Allen's offer and retire. Allen is not related to Marie and has good credit. Given these facts, which transfer method should be used to transfer the business to Allen? A) Grantor Retained Annuity Trust B) Self-Canceling Installment Note C) Private Annuity D) Installment Sale

D) Installment Sale Marie would sell the business to Allen by utilizing an installment sale and would charge a reasonable rate of interest. Because Allen would not have to pay the full sale price at the date of the transfer, he would not need to have all of the funds necessary at that time. Statement (A) is incorrect because GRATs are used to gift property, not for a sale. Statements (B) and (C) are incorrect because Allen is not related to Marie and she would not have any reason to enter into a SCIN or Private Annuity which may inequitably benefit Allen and possibly leave her with less than the fair market value of the business.

Which of the following is not a transfer cost associated with estate planning? A) Document preparation B) Attorney's fees C) CPA's fees D) Insurance premiums

D) Insurance premiums Insurance premiums are not a transfer cost associated with estate planning. All of the other answers are costs associated with estate planning.

Maud is a single 56-year-old woman who owns Shoe Designs, LLC. Her net worth is about $450,000 and she has not done any estate or business succession planning. 20 years ago, she had a falling out with her only daughter, Lena, and they have not had any contact since then. She has no other family and wants to donate her estate to Blogs for Dogs, a non-profit dog shelter. Which of the following statements would be correct if Maud died tomorrow? A) Finding a successor to take over her business will be a relatively simple task for her executor. B) Since she has no close family and her net worth is under the federal exemption amount, she does not need to do any estate planning. C) Maud's wish to donate her estate to charity can be accomplished through the state's intestacy laws. D) Lena will likely inherit all of Maud's property and no assets will go to charity.

D) Lena will likely inherit all of Maud's property and no assets will go to charity. Since Maud has not executed any estate planning documents, his estate will transfer via state intestacy laws. When an individual is not married, the individual's children are generally the next in line to inherit under state intestacy laws, even if they do not have a close relationship. Option (A) is incorrect because the ability to quickly sell a closely held business for fair market value is usually difficult and time-consuming unless prior arrangements have been made. Option (B) is incorrect because estate planning is necessary even for small estates. She will need to create a will to document her wish to transfer her assets to charity and for her advance medical planning. Option (C) is incorrect because charity and non-relatives do not stand to inherit any property under the state's intestacy laws.

Which of the following statements are correct regarding the taxable estate calculation? A) Funeral expenses and administrative expenses are deducted from the adjusted gross estate to calculate the taxable estate. B) Accounting and legal fees are deductible from the adjusted gross estate to calculate the taxable estate. C) Assets transferred to a surviving spouse are excluded from the gross estate. D) Losses from property that declined in value before the decedent's death are not deductible on the estate tax return.

D) Losses from property that declined in value before the decedent's death are not deductible on the estate tax return. Loss in property value is not deductible on the estate tax return, so loss property should be sold and the loss can be applied on the income tax return instead. Statements (A) and (B) are incorrect because those expenses are deducted from the gross estate to calculate the adjusted gross estate. Statement (C) is incorrect because assets transferred to a spouse are included in the gross estate and deducted from the taxable estate via the marital deduction.

Which of the following tasks could be performed by a financial planner who is not a licensed attorney or accountant? A) Drafting wills, trust documents, and powers of attorney B) Providing complex tax advice on the sale of an asset C) Assessing the fair market value of a small business D) Making investment recommendations and assisting with investment decisions

D) Making investment recommendations and assisting with investment decisions The financial planner generally assists in collecting investment decisions and making recommendations. If the financial planner is also a licensed attorney or a certified public accountant (CPA), the financial planner may take on the role of the attorney or CPA as well.

Elizabeth, a widow, has decided to set up trusts for each of her four grandchildren to take advantage of the generation-skipping transfer tax exemption. In 2021, she gives each grandchild $280,000. If Elizabeth has not made any previous taxable gifts, on what amount will she owe gift tax? A) $266,000 B) $280,000 C) $1,060,000 D) None

D) None The $11,700,000 GSTT exemption (2021) will fully cover her $1,060,000 taxable gifts to her grandchildren [($280,000 - $15,000) x 4].

Reese donated $100 to her church and $300 to the United Way. Which of the following is true regarding her contribution to the charitable organizations? A) Reese must file IRS Form 8283. B) Both her church and the United Way are required to send a confirmation of the contribution to Reese. C) Only her church is required to send a confirmation of the contribution to Reese. D) Only the United Way is required to send a confirmation of the contribution to Reese.

D) Only the United Way is required to send a confirmation of the contribution to Reese. Statement (A) is incorrect because IRS Form 8283 must be filed whenever the aggregate total of all noncash contributions exceeds $500. Statements (B) and (C) are incorrect because contemporaneous written acknowledgment by the donee organization is only required when an individual contributes cash or property valued at $250 or more. Therefore, Reese's church is not required to send a confirmation of Reese's donation.

Which of the following transfers would result in gift tax? A) Bob gifts $11,000 to his daughter, Barbie. B) Elroy gifts $50,000 to his wife, Elizabeth, who is a U.S. citizen. C) Adam gives his favorite employee, Aaron, a new car at Aaron's retirement worth $20,000. D) Pete transfers $20,000 to his ex-wife, Patricia. Pete and Patricia were divorced five years ago.

D) Pete transfers $20,000 to his ex-wife, Patricia. Pete and Patricia were divorced five years ago. Statement (A) would not result in gift tax because the gift does not exceed the annual exclusion. Statement (B) is incorrect because a person can gift an unlimited amount to his or her spouse, who is a U.S. citizen, without incurring gift tax. Statement (C) is incorrect because transfers in a business setting are presumed to be compensation. For Statement (D), if Pete had transferred $20,000 to Patricia pursuant to a divorce decree, there would be no taxable gift, but transfers to an ex-spouse five years after the divorce is final are not considered "transfers pursuant to a divorce decree."

Uncle Joe died recently. In Uncle Joe's will, he left his house to his favorite niece, Rachel, and his car to his other niece, Margaret. He did not use a residuary clause. Uncle Joe also owned a life insurance policy on his own life and named his two nieces the beneficiaries of the policy. He also owned a 401(k) plan without a listed beneficiary. Which of the following statements is correct? A) All assets will be transferred via the will. B) All assets will be transferred via the state's intestate probate laws. C) Some assets will be transferred via the will, and the remaining assets will transfer by operation of contract. D) Some assets will be transferred via the will, some by contract, and some by the state's intestate probate laws.

D) Some assets will be transferred via the will, some by contract, and some by the state's intestate probate laws. The house and the car will transfer under the will. The life insurance policy will transfer by contract outside the probate process because of the named beneficiaries. The 401(k) plan will transfer to Uncle Joe's probate estate because there is no listed beneficiary. Since the will does not cover the 401(k) plan, the asset will transfer via the state's intestate succession laws.

Which of the following statements concerning the GSTT is not correct? A) Each individual can exclude up to $11,700,000 of transfers from GSTT. B) The GSTT is applied to a gift after the application of the annual exclusion. C) Gifts that are subject to GSTT can be split. D) The GSTT only applies to transfers in trust.

D) The GSTT only applies to transfers in trust. The GSTT applies to the transfer of any property to a skip person or an interest in trust for the benefit of a skip person. All of the other statements are true.

Which of the following is not a reason that the proceeds of a life insurance policy would be included in a decedent's gross estate? A) The proceeds of the policy are payable to the estate. B) The decedent transferred the ownership of the policy to his daughter six years before his death, but retained the right to change the beneficiary of the policy. C) The decedent transferred the ownership of the policy to his son six months before his death. D) The decedent transferred the ownership of the policy to his wife four years ago.

D) The decedent transferred the ownership of the policy to his wife four years ago. Statement (A) is incorrect because the proceeds of the policy would be included in the estate if the proceeds are payable to the estate. Statement (B) is incorrect because the decedent is considered to have an incident of ownership in the policy if he retains the right to change the beneficiary of the policy. Statement (C) is incorrect; under IRC section 2035, the proceeds of a policy transferred within three years of death are included in the gross estate of the transferor.

Which of the following is not true regarding taxation of trust income? A) Trust income is taxed to the trust if it is accumulated and taxed to the beneficiaries if it is distributed. B) Beneficiaries are liable for all income tax in a simple trust. C) A trustee with discretionary powers may be liable for tax on all trust income earned. D) The grantor is always liable for all tax on trust income.

D) The grantor is always liable for all tax on trust income. Liability for tax on trust income depends on whether the income was distributed or accumulated and whether or not the trust is a grantor trust.

Which of the following is not a requirement of the unlimited marital deduction? A) In order to claim a marital deduction, the decedent must have been married as of the date of his death. B) The surviving spouse must receive property through the estate. C) The surviving spouse must be a U.S. citizen. D) The gross value of qualifying property left to the surviving spouse is included in the marital deduction.

D) The gross value of qualifying property left to the surviving spouse is included in the marital deduction. Statements (A), (B), and (C) are all requirements of the unlimited marital deduction. Statement (D) is incorrect because only the net value, not the gross value, of qualifying property left to the surviving spouse is included in the marital deduction. The term "net value" for marital deduction purposes equals the gross value of the qualifying property left to the surviving spouse less any taxes, debts, or estate administration expenses payable out of the spousal interest.

Which of the following statements is correct regarding the applicable estate tax credit? A) When a decedent's taxable estate is less than the applicable estate tax credit equivalency, the estate must still file the 706. B) When too few assets pass to a decedent's surviving spouse, and as such, the decedent's taxable estate is greater than the applicable estate tax credit equivalency, the decedent's estate is said to be overqualified. C) An ABC Trust arrangement utilizes a General Power of Appointment Trust, a QTIP Trust, and a GST Trust to maximize the use of a decedent's applicable estate tax credit. D) The portability provision allows an unused credit amount to be utilized by a surviving spouse.

D) The portability provision allows an unused credit amount to be utilized by a surviving spouse. Statement (A) is incorrect because Form 706 is only required if a taxable estate exceeds the federal exemption amount. Statement (B) is incorrect because the estate would be underqualified. Statement (C) is incorrect because ABC trust structures require the use of Bypass trusts, not GST trusts.

All the following statements regarding the probate process are correct EXCEPT: A) Distribution of probate assets involves more time and expense due to the involvement of the probate court system B) The probate process involves retitling assets to the appropriate heirs C) The probate process provides a window for creditors to raise claims against the estate. D) The probate process allows assets to be transferred privately.

D) The probate process allows assets to be transferred privately. Certain information is required to be filed with the probate court, and that becomes public information. Options (A), (B), and (C) are true statements.


Conjuntos de estudio relacionados

Chapter 3: Body, Diagnosis, Equipment, Instruments, and Treatment

View Set

Chapter 8: Current and Contingent Liabilities

View Set

GACE: Major American Authors & Summary of Works

View Set

Developmental Psychology Exam Questions

View Set

Exam 3- Units 11-14- Nursing 121

View Set

Chapter 16 Oceans and Coastlines

View Set

TestOut Network Pro 6.1.8 Practice Questions

View Set

Carlos explora Lima, Perú - Productos, lugares, y/o personas

View Set